Está en la página 1de 48

w

U
o
z

,--

01 .

Semiologa urolgica
y definiciones

1.l.

Definiciones

1.2.

Diagnstico diferencial
de la hematuria macroscpica

02.

01
01
02

Infecciones

VI

Urolitiasis

3.l.

Epidemiologia

3.2.

Manifestaciones clnicas

y su manejo agudo
Evaluacin y tratamiento

10

3.3.

de la litiasis renal

11

09
09

04.

Tumores renales

03

4.1.

Carcinoma de clulas renales


(adenocarcinoma renal, hipernefroma)

15

4.2.

Otros tumores

17

05.

Hiperplasia

del tracto urinario.


Cistitis intersticial

03.

2.l.

Patognesis y etiologa

03

2.2.

Diagnstico

04

2.3.

Diferentes ITU y su tratamiento

04

2.4.

Tuberculosis genitourinaria

07

2.5.

Cistitis intersticial

07

y carcinoma prosttico

15

19

S.l.

Hiperplasia prosttica benigna

19

5.2.

Carcinoma prosttico

21

06.

Carcinomas
del tracto urinario

26

6.l.

Carcinoma vesical

26

6.2.

Tumores del tracto urinario superior

28

07.

Tumores testiculares

31

7.1.

Etiologa y epidemiologa

31

7.2.

Anatoma patolgica

32

7.3.

Clnica

32

7.4.

Diagnstico

32

7.5-

Diagnstico diferencial

33

7.6.

Tratamiento

34

08.

Trasplante renal

36

8.l.

Indicaciones

36

8.2.

Contraindicaciones

36

8.3.

Complicaciones

37

09.

Uropata obstructiva

38

9.l.

Caractersticas

38

9.2.

Clnica

38

9.3.

Diagnstico

39

9.4.

Tratamiento

39

1 0.

Disfuncin erctil

40

10.1.

Introduccin

40

10.2.

Prevalencia

40

10.3.

Etiologa

40

10.4.

Factores de riesgo

40

10.5-

Diagnstico

41

10.6.

Tratamiento

41

Bibl iografa

42

VII

01

SEMIOLOGA UROLGICA
y DEFINICIONES
Aspectos esenciales

Orientacin

MIR
Este tema no se ha preguntado
en el MIR de forma directa.
Puede ayudar a obtener una
visin general de la materia
ya asociar algunos hallazgos
a patologras concretas, pero
no se debe emplear en l
demasiado tiempo.

GJ

La causa ms frecuente de hematuria microscpica es la litiasis (poblacin general, ambos sexos).

III

la causa ms comn de hematuria microscpica en varones de ms de 50 aos es la hiperplasia benigna


de prstata.
la hematuria con cogulos indica un problema urolgico.
la causa ms habitual de hematuria es la cistitis hemorrgica, pero 10 primero a descartar es el tumor uro

IIJ

leJia!.

Los hemates dismrficos en el sedimento orientan a nefropala de origen glomerular.

1 . 1 . Defin iciones
Hematuria microscpica: presencia de ms de 5 hemates por campo. La causa ms frecuente en ambos sexos

es la litiasis. La causa mas comn en varones mayores de 50 aos es la hiperplasia benigna de prstata.

Hematuria macroscpica: orina de aspecto rojizo a simple vista debido a la presencia de ms de 50 hemates

por campo. En los pacientes fumadores, en ausencia de otros sntomas, se debe sospechar tumor urotelial.
Piuria: presencia de ms de 10 leucocitos por campo. Altamente nespecfica, pero en presencia de sntomas

urinarios, hay que sospechar infeccin.

Sndrome miccional: presencia de polaquiuria (aumento en la frecuencia miccional), urgencia miccional (ne

cesidad imperiosa e irrefrenable de orinar) y disuria (molestias urinarias inespecficas referidas como ardor,
escozor, etc.).
Incontinencia urinaria: prdidas involuntarias de orina (Tabla 1 l. Existen cuatro tipos principales:
INCONTINENCIA

INCONTINENCIA

DE URGENCIA

DE ESFUERZO

Urgencia (deseo rwpentIno de ortnar)

Si

No

Aumento de laheuencla mlcdonal

Si

No

Copoc:ldod de __ .. bollo despus de _ 01 deseo de orinar

No

SI

_Ir"boIio_la_

sr

Generalmente no

Escape _la lICtMdad fIsIca

No

Si

Abundante. si se produce

Generalmente escasa

SINTOMAS

eanddod de orIno ....poda


.
en coda opIlIOdlo de IncontI"'''cla

Tabla 1. Diagnstico diferencial de la cl nica de incontinencia urinaria femenina

Continua: de da y de noche, en todas las posiciones. La causa ms frecuente es la fstula urinaria (en pacientes
con antecedentes quirrgicos previos) y la segunda, el urter ectpico (que es la causa ms frecuente en nias).
De esfuerzo: se desencadena con el aumento de presin abdominal (al rer, toser, cargar con peso). Ge
?

Preguntas

- No hay pregunta s MIR


representativas.

neralmente se produce por un dficit de soporte de la musculatura perineal (por ello es recomendable
revisar los antecedentes obsttricos, pacientes obesas, pacientes aosas, ete.).
De urgencia: el paciente siente ganas de orinar, pero no le da tiempo a l legar al bao (provocadas por
contracciones involuntarias del msculo detrusor).

Manual CTO de Medicina y Cirugfa, 8.a edicin


Mixta: generalmente es una combinacin de las dos anteriores.
Paradjica: escape de orina debido a la sobreclistensin vesical.
El ejemplo caracterstico es el paciente prosttico con retencin
urinaria. La presin i ntravesical supera la presin de cierre del

1 .2. Diagnstico d i ferencia l


d e l a hematuria macroscpica

esfnter uretral, producindose un escape de orina paradjico


(no puede orinar y, sin embargo, se le escapa la orina).

RECUERDA

los cilindros hemticos aparecen en las glomerulonefritis que producen


sndrome nefrtico, como en la postinfecciosa.

Segn el momento de aparicin:


Inicial: sangrado uretral o prosttico.
Final: sangrado del cuello vesical.
Total: vesical o del tracto urinario alto. Se debe recordar que un
sangrado importante de cualquier parte del aparato genitourina
rio puede provocar hematuria total.
Hematuria con cogulos: indica un problema urolgico. La cau

Enuresis: prdidas de orina exclusivamente durante el sueo. Si el

sa ms frecuente en mujeres es la cistitis hemorrgica, aunque la

nio es mayor de 6 aos, debe ser estudiado.

primera causa a descartar es una neoplasia urotelial, mxime en el

Crisis renoureteral: dolor lumbar frecuentemente irradiado a geni

paciente fumador.

tales, de carcter agudo, cuya intensidad no se modifica por los

Hematuria por nefropata mdica: no suele tener cogulos, y puede

cambios pastura les, y que se suele acompaar de nuseas, vmitos


y malestar general. Es muy poco frecuente que sea bilateral.

eritrocitarios o de hemates dismrficos en el sedimento urinario.

ir acompaada de cierto grado de proteinuria, as como de cilindros

02.
INFECCIONES DEL TRACTO URINARIO.
CISTITIS INTERSTICIAL

Orientacin

MIR
la causa ms frec uente de infeccin del tracto urinario OTU) es Escherichia coli, tanto a nivel comunitario
como nosocomial.

Este tema es el ms importante


de esta asignatura. Se debe

El origen ms frecuente de uretritis es Chlamydia lrachomalis.

conocer muy bien, pues dos


o tres preguntas son habituales
en el examen.

la causa habitual de orquiepididimitis depende de la edad: Chlamydia y gonococo si es menor de 35 aos;


enterobacterias, si es mayor de esa edad.

El estudio de las

preguntas de otros aos suele

ser de gran ayuda, ya que son


baStlnle repetitivas. No se

la causa ms frecuente de absceso renal en el UDVP es Staphylococcus aureus.

debe bajar la guardia con l a

El diagnstico definitivo de ITU es microbiolgico: ms de '05 UFClm1. No obstante, este criterio vara con
el sistema de recogida.

tuberculosis genitaurinaria n i
e n la cistitis intersticial. Hay
que formar una imagen mental
tpica para reconocerlas en

Si se recoge la muestra urinaria mediante puncin suprapbica, cualquier nmero de bacterias es significativo.

caso clnico, que es como


suelen preguntarlas.

la bacteriuria asintomtica se trata en gestantes, menores de 5 aos, inmunodeprimidos, previamente a la


ciruga urolgica, o si la especie implicada es Proteus.

Los sistemas de drenaje cerrados son preferibles a los abiertos, pues la lasa de infeccin es menor.

La infeccin del tracto urinario (lTU) puede clasificarse de varias formas. Se puede hacer una divisin anatmica
entre las ITU altas (infecciones renales) y las ITU bajas (cistouretritis, prostatitis). Asimismo, la clasificacin pue
de basarse en la asociacin o no de complicaciones. Una ITU no complicada es un cuadro clnico caracterizado
por la presencia de escozor miccional, urgencia y frecuencia, acompaado o no por hematuria terminal, dolor
hipogstrico, y ms raramente, febrcula. Dentro de este grupo se podran incluir las pielonefritis no complica
das, que se presentan como cuadros febriles con hipersensibilidad en fosa lumbar, fiebre, nuseas o vmitos, y
sin los factores que convierten la [TU en "complicada", como son: presencia de catteres, uropata obstructiva,
reflujo vesicoureteral, anomalas anatmicas, insuficiencia renal o trasplante renal. La ITU en el varn debe
considerarse esencialmente "complicada" de entrada.
La reaparicin de una infeccin tras el tratamiento puede deberse a reinfeccin o recidiva. El primer trmino expre
sa la infeccin nueva por un germen distinto al inicial, mientras que recidiva indica infeccin por el mismo germen.
Esta ltima es mucho ms infrecuente que la reinfeccin y puede estar ocasionada por litiasis infectiva, prostatitis
crnica, fstulas vaginales o intestinales, divertculos vesicales infectados, cuerpos extraos, necrosis papilar infec
tada y otras causas que generan un reservario de microorganismos que difcilmente se eliminan con el antibitico.

2 . 1 . Patognesis y etiolog a
III

Existen tres posibles vas por las que los microorganismos pueden alcanzar el tracto urinario: hematgena,

Preguntas

- MIR 08-09,98
- MIR 05-06, 230
-MIR 03-04, 78,8 4
- MIR 02-03, 1 34,1 74
-MIR OJ-02 , 103, 1 06
-MIR OO-01F,1 34, 1 45
- MIR 99-00, 1 35
- MIR 99-00f,1 1 9
- MIR 98-99F, 1 1 8
- MIR 97-98, 26,206, 2 1 5

linftica y ascendente. La va l inftica carece de i mportancia real. La diseminacin hematgena tampoco es


ferecuente. La ms comn es la ascendente i niciada en la uretra. Probablemente por esta razn es mucho ms
habitual la ITU en mujeres, dado que su uretra es muy corta y ancha, y por ello favorece el paso de microorga
nismos hacia niveles ms altos del TGU.
Otro dato que apoya la importancia de la va ascendente es la frecuencia de i nfeccin tras el cateterismo
uretral, que es del 1 % en los pacientes ambulantes, y en tres o cuatro das alcanza a casi la totalidad de los
pacientes sondados con sistemas de drenaje abiertos. En los enfermos hospitalizados, el riesgo de i nfeccin
alcanza un 5% por cada da de sondaje, incluso con sistemas cerrados (MIR 0 1 -02, 103).

Manual CTO de Medicina y Ciruga, 8.a edicin


Una vez que las bacterias han alcanzado el tracto urinario, tres factores
determinan el desarrollo de la infeccin:
La virulencia del microorganismo.
El tamao del inculo.
Los mecanismos de defensa del husped.

2.3. Diferentes ITU y su trata m ie nto


En el tratamiento de la ITU lgicamente es fundamental el empleo de
antimicrobianos. El nmero de stos empleado es elevado y las pautas

La mayora de las infecciones en la comunidad estn producidas por


grmenes gramnegativos, principalmente E. coli (MIR 05-06, 230), res

de tratamiento muy variables. A continuacin, se repasarn las opcio


nes teraputicas segn el tipo de ITU a la que uno se enfrente.

ponsable del 85% y, en menor proporcin, Proteus, Klebsie/la o Pseu


domonas (MIR 97-98, 2 1 5). Entre los grampositivos, nicamente el Sta
phylococcus saprophyticus tiene relevancia, produciendo el 10- 1 5%

Bacteriuria asi ntomtica

de las ITU en mujeres jvenes (segundo germen ms frecuente en esta


poblacin).
Definida como bacteriuria significativa (105 UFC/ml) en a l menos dos
Alrededor del 30% de las mujeres con clnica miccional presentan re

urocultivos con el mismo germen, tomados con una semana de dife

cuentos menores de 105 colonias por mililitro ( 1 05 UFC/ml); de stas,

rencia en ausencia de sntomas. la bacteriuria asintomtica no debe

tres cuartas partes presentan piuria; en el resto, existen pocos datos

tratarse salvo en los casos en los que conlleva un riesgo de i nfeccin

que demuestren infeccin, y en general se tratan segn la clnica. En


la orina de las pacientes sintomticas con piuria, se pueden encontrar

clnica o dao orgnico, como ocurre en nios menores de 5 aos,

(considerndose infeccin activa) recuentos ms bajos (1 02- 1 04) de

en el embarazo (MIR 08-09, 98 ; M I R 03-04, 78), en pacientes inmu

los patgenos habituales. En otras ocasiones, el cuadro se justifica por


la presencia de uretritis causada por N. gonorrhoeae o C. trachomatis.

tengan o no patologa urolgica asociada. Asimismo, debe ser tratada


nodeprimidos, como profilaxis previa a una ciruga urolgica y en los

casos de bacteriuria por Proteus (MIR 02-03, 1 34) (Tabla 2).

El papel patgeno de grmenes como U. urealyticum o Mycoplasma

hominis est mal definido, ya que se desconoce su potencial como

uropatgenos aislados (MIR 99-00F, 119).


En las i nfecciones nosocomiales, los grmenes gramnegativos conti
nan siendo los ms frecuentes. Si bien E. coli es el ms habitual, su
frecuencia desciende hasta el 50% y adquieren mayor importancia

Embarazadas
Inmunodeprimidos
Previamente a ciruga urolgica
Bacteriuria por Proteus
Tabla 2. Bacterruria asintomtica: indicaciones de tratamiento

Proteus, Klebsiella, Pseudomonas, Enlerobacler y Serratia (MIR 03-04,

84). El 25% restante est ocasionado por grmenes grampositivos como


estreptococos y estafilococos. Candida albicans puede aparecer prin
cipalmente en pacientes diabticos, cateterizados o con tratamientos
antibiticos prolongados.

D RECUERDA

Proteus es intrnsecamente resistente ti las nitrofurantonas, ya que al


caliniza la orina gracias a su ureasa, y este grupo de antibiticos nica
mente es til en medio cido.

la afectacin del tracto urinario superior parece tambin producirse


por ascenso de los grmenes a lo largo del urter. La diferenciacin,
aunque poco especfica, se debe basar en los hallazgos clnicos (fiebre,

En el caso de los pacientes sondados permanentemente, la presencia

dolor lumbar, escalofros) y analtica elemental ( leucocitosis, velocidad

de bacteriuria asintomtica no es una indicacin de tratamiento y,

de sedimentacin alta).

actualmente, incluso es dudosa la recomendacin clsica de empleo


profilctico de algn antibitico, previo a la sustitucin del catter, a
fin de contrarrestar la posible diseminacin hematgena del germen

2.2. D i a g nstico

producida por la manipulacin (las ltimas guas clnicas ya no lo reco


miendan). S es, sin embargo, i ndicacin de tratamiento la bacteriuria
persistente a los 3-5 das de haber retirado una sonda vesical. En aque

E l d i agnstico de ITU, adems de la clnica, se define por el cultivo


de orina. Dado que es frecuente el crecimiento de bacterias que han

llos pacientes en los que la sonda no pueda ser retirada, el tratamien


to de las bacteriurias asi ntomticas no suele ser efectivo, y puede dar
lugar a seleccin de cepas resistentes. En estos pacientes slo se debe

contaminado las muestras, se utiliza u n criterio estadstico sobre la

iniciar tratamiento si presentan alto riesgo de desarrollar bacteriemia o

base del recuento de colonias del urocultivo, considerando como

si la bacteriuria se hace sintomtica.

significativo clsicamente el crecimi ento de ms de 1 05 colonias por


m i l i l itro (MIR 97-98, 206). En determinadas circunstancias, recuen

En el resto de los casos, nicamente con la concurrencia de factores

tos de colonias menores pueden ser suficientes: recuentos de 1 03

particulares, se debe tratar la bacteriuria, y siempre sobre la base del

UFC/ml en mujeres sintomticas, ms de 1 04 en pielonefritis c l nicas

estudio de sensibil idades (MIR 98-99F, 1 1 8).

o en varones, y ms de 1 02 en muestras de cateterismos l i mpios


o cualquier recuento, si se recoge mediante puncin-aspiracin su
prapbica (MIR 97-98, 26). Cifras mayores de 105 U FC/ml pueden

ITU baja en mujeres

igualmente reflejar contaminacin, principalmente si crecen dos o


ms especies.
Puede realizarse un tratamiento convencional de siete das o bien u n
En el adulto, la presencia de piuria (ms de 1 0 leucocitos/mm3) se rela
ciona estrechamente con la ITU en presencia de sntomas, no as en el

curso corto e n monodosis o en rgimen d e tres das. L a ventaja d e stos


es el menor coste econmico y la menor incidencia de efectos adver

nio, en el que puede acompaar a los cuadros febriles.

sos. Su desventaja es la mayor incidencia de recurrencias tempranas, a l

urologa

no afectar apenas a los reservorios vaginal e intestinal de uropatgenos.


Aun con todo, por las ventajas mencionadas, la pauta preferida actual

El antibitico empleado se seleccionar, por supuesto, sobre l a


base del cultivo y d e l antibiograma, y cuando s e i n icie d e forma

mente es el tratamiento de tres das.

emprica, habr que tener en cuenta factores que orienten hacia

Los antibiticos de eleccin son el cotrimoxazol, las fluoroquinolonas

el germen causante: mayor incidencia de Pseudomonas en perso


nas diabticas y enfermos de UVI, estafilococo en adictos a drogas

y la amoxicilina-cido clavulnico, fosfomicina o n itrofurantona.

parenterales (MIR 03-04, 84), Proteus en pacientes con litiasis in


fectiva, presencia de sondas, catteres, tratamientos antibiticos

En mujeres embarazadas se recomiendan las pautas largas de trata

previos, etc.

miento (siete das), evitando el uso de sulfamidas al final del embarazo


por el riesgo incrementado de kernicterus, y el empleo de quinolonas

En la evaluacin del paciente con pielonefritis y mala respuesta a tra

por el dao producido sobre el cartlago de crecimiento fetal. Tampoco

tamiento inicial, es recomendable la realizacin de una ecografa para

se emplearn pautas cortas en caso de sospecha de pielonefritis, pre

descartar obstruccin o litiasis (M I R 00-01 F, 134).

sencia de clculos o anomalas de la va urinaria, o bien infecciones


previas por microorganismos resistentes a los antibiticos.

ITU en varones

ITU recurrente

Cualquier ITU en varn debe considerarse como complicada inicial


mente ya que hay que asumir que existe afectacin del tejido prost
tico, renal o que existen problemas concomitantes como obstruccin

Aparicin de cuatro o ms episodios al ao. Se puede realizar profi

urinaria, litiasis o malformaciones urolgicas. Por todo ello, el trata

laxis con cotrimoxazol O una fluoroquinolona (en funcin de la sensi

miento debe ser ms prolongado (mnimo una semana), no siendo ade

bilidad del germen aislado en el ltimo episodio) en dosis nica, das


alternos, durante seis meses. Si tras la retirada se presentaran nuevas
recurrencias, puede reinstaurarse el tratamiento durante periodos ms
prolongados (1-2 aos). Es aconsejable la ingesta abundante de agua

cuados los cursos cortos de tratamiento.

Prostatitis

y realizar micciones frecuentes y cumplir una serie de reglas bsicas


higinico-dietticas.
La infeccin aguda del tej ido prosttico se presenta como un cuadro
Si los episodios tienen relacin con el coito, se puede administrar un
comprimido de cotrimoxazol o una quinolona despus del mismo. En
mujeres posmenopusicas, el tratamiento con estrgenos tpicos vagi

rectal, la prstata aparece muy dolorosa e inflamada. El germen ms


habitual es E. coli. Durante la inflamacin aguda, los antibiticos pe
netran adecuadamente, pero una vez que sta cede, la penetracin es

nales disminuye la frecuencia de i nfecciones.

RECUERDA

sptico con afectacin general del paciente, fiebre elevada, sndrome


miccional, artromialgias y dificultad miccional (Tabla 3). En el examen

ms pobre. Por ello, se deben utilizar cursos largos de tratamiento (3-4

Staphylococcus saprophylicus

se ha relacionado con

ITU

jvenes sexualmente activas.

semanas) para i n tentar evitar la persistencia de focos que den pie a una
prostatitis crnica. Entre los antimicrobianos empleados, las fluoroqui

en mujeres

nolonas son las que mejor difunden al tejido prosttico .

RECUERDA

Pielone!ritis aguda no complicada

En pacientes con SIDA, Cryptococcus neoformans puede ser una causa


de prostatitis, ya que se elimina a travs de la orina.

En los casos de gravedad leve-moderada, puede plantearse terapu


tica oral con cotrimoxazol (en desuso en nuestro medio por el ele
vado ndice de resistencias), fluoroquinolonas o B- Iactmicos. En
pacientes graves u hospita l i zados es preciso tratamiento parenteral,

La prostatitis crnica bacteriana suele presentarse como molestias


perineales o genitales, sntomas i rritativos (polaquiuria, tenesmo, es

y el espectro de antimicrobianos incluye ampicilina (enterococo),


ureidopen icilinas (Pseudomonas), cefalosporinas de segunda o ter

cozor) y episodios de ITU recurrentes causados por el mismo organis

cera generacin, e incluso aminoglucsidos. Nunca se emplearn

campo de gran aumento, y macrfagos que contienen cuerpos ovales

pautas cortas.

grasos.

ETIOLOGIA

ogudo

Prostadtls
crnica

crnica

no*-no

ClINICA

mo. En el lquido prosttico se evidencian ms de 10 leucocitos por

H 'ITU

CULTIVO ORINA

LIQUIDO PROSTATlCO

CULTIVO LIQUIDO
PROSTATICO

TRATAMIENTO

Cotrimoxazol,
fluoroquinolonas
4 semanas

E.coH

Cuadro sptico

E.coJ/

Irritativo con
reagudizaciones, sin
fiebre ni leucocitosis

+/-

>

10 leucocitos/campo

Cotrimoxazol,
fluoroqulnolonas
6-12 semanas

Ureaplasma

Cronicidad,
empeoramiento

>

10 leucocitos/campo

Doxicidina

Desconocida

Oscilante

< 10 leucocitos/campo

a-bloqueantes
Relajantes musculares

Myeoplasma

Nunca hacer masaje prosttico ni sondaje

Tabla 3. Oiagnstko diferencial de las prostatitis

Manual CTO de Medicina y Ciruga, B." edicin


El tratamiento debe estar guiado por los cultivos, tanto de orina como
de fluido obtenido por masaje prosttico, y prolongarse entre 4 y 1 6
semanas. Cuando se encuentra a un paciente con datos de prostatitis
crnica y signos inflamatorios en el lquido prosttico, pero sin historia
documentada de ITU y con cultivos negativos, el cuadro se denomina
prostatitis no bacteriana. En ocasiones, el responsable puede ser U.

urealylicum o M . hominis, pudiendo ser tratados estos casos con doxi

ciclina o eritromicina, sobre esta sospecha.


Se denomina prostatodinia a un cuadro clnico similar donde predo
minan las molestias perineales o genitales con cultivos negativos y
menos de 1 0 leucocitos por campo en el lquido prosttico. Su cau
sa es desconocida y el tratamiento difcil, emplendose actualmente
p-bloqueantes o relajantes musculares como terapia inicial.

Orqu iepididimitis
En varones adultos menores de 3 5 aos e s considerada, en el plano
terico, una enfermedad d e transmisin sexual, siendo los agentes
ms frecuentes Chlam ydia trachomatis y Neisseria gonorrhoeae (Ta
bla 4). Por encima de 35 aos, los microorgan i smos ms frecuentes
son las enterobacterias. El tratamiento puede l l evarse a cabo con las
sigui entes pautas: 1 ) quinolonas, 2) ceftriaxona en dosis nica i . m .
( 1 2 5-250 mg) ms 1 0 das de doxiciclina ( 1 00 mgl1 2 h/7 das), se
aplicar esta pauta en aquellos casos en los que se sospeche ETS
(MIR 99-00, 1 35).

Absceso renal
Los abscesos medulares o corticales suelen proceder de un foco de pie
lonefritis contiguo O de diseminacin hematgena de s. aureus/ proce

dente de focos cutneos en sujetos adictos a drogas por va parenteral .

E l urocultivo e n este ltimo caso puede ser negativo. El diagnstico

ms fiable se realiza mediante TC Deben tratarse con antibiticos

El diagnstico es similar al absceso renal, y su tratamiento pasa por

por va intravenosa y, dependiendo del tamao y de la evolucin,

el drenaje percutneo o quirrgico, con la adecuada cobertura anti


bitica.

se hace obligatorio el drenaje mediante puncin percutnea


gicamente.

quirr

ITU asociada a catteres


Absceso pe,i"enal

Se localiza entre la cpsula renal y la fascia de Gerota. Lo ms fre

La ITU es la infeccin hospitalaria ms frecuente, y los catteres urina

cuente es que un absceso cortical se abra a este espacio, pero pue

rios la principal fuente de sepsis. Se calcula que el 1 % de cateterismos

d e ocurrir tambin por diseminacin hematgena. El germen ms

ambulatorios transitorios sufren una ITU posterior y que la mayora de

D RECUERDA

S. oureus es tambin la causa ms frecuente


de endocarditis Infecciosa.

ENFERMEDAD

ETIOLOGIA

enfermos con catter permanente presentan una bacteriuria significati

S. aureus en los casos

va al cuarto da de su colocacin. Esta bacteriuria puede hacerse sinto

de diseminacin he

mtica en forma de cuadros de cistitis, hematuria o episodios febriles,

matgena (Figura 1).

muchas veces autolimitados.

DIAGNSTICO

LESiN TIPICA
Asintomtica ()
Exudacin uretral matutina (1;)
Epididimoprostatitis, salpingitis, sndrome
Fitz-Hugh-Curtis, gonococemia diseminada
(dficit C5-C.. menstruacin, embarazo,
auxotipo AHU)

Contacto < 5 dlas


Gram de exudado cervical
Cultivo en medio de Thayer-Martin

Ceftriaxona o
espectinomicina (no en
farngeas)Ciprofloxacino

Chlomydio trochomotis,
Ureoplosmo ureolyticum

Similar a las UG, pero con menos signos

Contacto 7-15 dias. Excluir gonorrea


por Gram y cultivo. C. inclusinGiemsa IFD, medios celulares

Tetraciclinas o macrlidos

y sntomas
Epldidimitis, proctitis, cervicitis, EIP

Tabla 4. Diagnstico diferencial de las uretritis

TRATAMIENTO

Neisserio gonorrhoeoe
Uretritis gonoccIco

lInIIrItIs no __

frecuente es E. coli, y

urologia

Entre los factores que aumentan el riesgo de ITU asociada a catter


urinario se pueden enumerar: 1) sexo femenino, 2) edad avanzada, 3)

Clnica

mala tcnica de sondaje, 41 sistemas de drenaje abiertos y 51 falta de


higiene local.

Los hallazgos clnicos son escasos. En el 70% de los pacientes, los sn


tomas son leves. Lo ms frecuente es la aparicin de microhematuria,

Entre los antibiticos disponibles, parece que las quinolonas son los

dolor vago en flanco o clico renal. La afectacin vesical, sin embargo,

que mejor eliminan la pelcula biolgica de los catteres infectados,

s produce sintomatologa florida con un sndrome cisttico rebelde,

favoreciendo as el tratamiento de la infeccin; en cualquier caso, ste

donde la polaquiuria (secundaria a la disminucin de la capacidad

nicamente se recomienda si existe sintomatologa o en el momento de

vesical) es lo ms llamativo. En varones, es frecuente la aparicin de

la retirada del catter, por el mayor riesgo de ITU sintomtica y sepsis.

una orquiepididimitis crnica que no responde a la terapia habitual.

2.4. Tu berc u l os is g e n itou r i n a ri a

aparece piuria cida con urocultivo negativo. La prueba de laborato


rio ms importante es el cultivo de M. tuberculosis en medio selectivo

En el 90% de los pacientes, el anlisis urinario es anormal. Tpicamente

(Lowenstein), ya que los medios de tincin rpida (Ziehl, auramina),


aunque vlidos, pueden dar falsos positivos por contaminacin con M.
Generalmente est ocasionada por Mycobacterium tuberculosis. El apa

smegmatis (MtR 02-03,

174).

rato genitourinario es el sitio ms frecuente de afectacin extrapulmonar


(tras la adenitis tuberculosa). Un 5% de los pacientes con tuberculosis
activa presentan afectacin del tracto genitourinario (Figura 2).
Tuberculosis
miliar

Diag nstico
El cultivo en medio de Lowenstein es positivo en el 90% de los pa
cientes con enfermedad activa, aunque deben obtenerse, al menos, tres
muestras de das diferentes para mejorar la sensibi lidad, ya que el paso
de bacilos a orina no es constante. Actualmente, lo ms rentable es rea
lizar una PCR de orina en busca del ARN del bacilo.
Radiolgicamente, el 90% de los pacientes presentan urogramas altera

Amputacin
catieial

Pionefrosis

dos. El hallazgo ms sugestivo es la presencia de cavidades que comu


nican con el sistema colector. Inicialmente estas cavidades son mnimas
y dan un aspecto "mordisqueado" a los clices. Segn la enfermedad
avanza, pueden encontrarse estenosis infundibulares, ureteropilicas, en

Trompa
Mierovejiga

Prstata

ms evolucionado de la enfermedad, el rin puede encontrarse anula


do, disminuido de tamao y con calcificaciones parenquimatosas.

y vesfeulas

Trata miento

Epididimitis

mente del de la tuberculosis pulmonar en cuanto a frmacos y periodo

seminales

Estenosis
ureteral distal

unin ureterovesical o vejigas pequeas de aspecto rgido. En el punto

El tratamiento mdico de la enfermedad activa no difiere sustancial


de tratamiento. Puede ser necesario el tratamiento quirrgico, depen
diendo de la complicacin asociada, generalmente estenosis de la va

Figura 2. Lesiones de la tuberculosis genitourlnaria

excretora e h idronefrosis. En caso de rin no funcionante por lesin


extensa del parnquima, puede ser precisa la nefrectoma.

Tras la inhalacin del bacilo, se produce una diseminacin hematgena


(primoinfeccin) con siembra de bacilos en ambos riones en el 90% de
los casos. Sin embargo, la enfermedad clnica generalmente es uni lateral.

11 :?n, isoniacida y pirazinamida son los tres antibiticos ms em


pleados en la tuberculosis.

El periodo de latencia entre la "siembra" y la enfermedad clnica oscila


entre 1 O Y 40 aos, afectando principalmente a pacientes por debajo de
los 50 aos. La lesin inicial microscpica se localiza en los glomru
los en forma de granulomas microscpicos. Al avanzar la enfermedad,

2.5. C istitis intersticial

se produce afectacin ms distal hasta la aparicin de una papilitis


necrotizante, momento en el cual ya puede existir paso de bacilos a la
va excretora donde, por procesos inflamatorios, ocasionar estenosis

Aunque no es u n cuadro infeccioso, s e incluye e n el presente captulo

a nivel de los infundbulos caliciales, pelvis y urter, con hidronefrosis

esta entidad inflamatoria vesical de origen desconocido. En este senti

secundaria. Las lesiones renales pueden cavitarse y calcificarse, y lle

do, se esgrimen dos teoras no demostradas: por un lado, la teora au

gar a producir una destruccin total del parnquima (fenmeno que se

toinmunitaria, y por otro, la de un dficit en el recubrimiento urotelial

denomina "rin mastic" ).

por glucosaminoglucanos.

Manual elO de Medici na y Ciruga, s.a edicin

Cl nica

litiasis o tumor vesical) apoyado en los hallazgos cistoscpicos suges


tivos: 1) petequias submucosas, principalmente trigonales, que apare

Suele presentarse en mujeres entre 30 y 70 ar1os, como un cuadro cis

cen al distender la vejiga (glomerulaciones), 2) lceras de Hunner. La

dolor suprapbico, acompaados en ocasiones de hematuria (20-30%)


(MIR 01-02, 1 06) .

u otra patologa, revela en algunos casos, un infiltrado intersticial de


mastocitos (MIR 00-01 F, 1 45).

ttico crnico en el que destacan disuria, polaquiuria con nicturia y

D RECUERDA

biopsia vesical, adems de descartar la presencia de carcinoma in situ

Trata miento

Existen muchas ms causas de sndrome cisttico: cistitis aguda, tuberculo


sis, carcinoma in situ, etc.

Aunque esta enfermedad raramente supone una amenaza para la vida de


la paciente, su morbilidad es elevada. Desgraciadamente, las diversas al

Diag nstico

ternativas de tratamiento nicamente pueden encaminarse a una mejora


sintomtica, en la mayora de los casos con resultados discretos; 1 ) dis
tensin hidrulica vesical, 2) amitriptilina oral, 3) instilacin con dime

El diagnstico es bsicamente por exclusin de otra patologa que


pueda ocasionM un cuadro similar (infeccin bacteriana, tuberculosis,

tilsulfxido [DMSOJ, 4) corticoides tpicos o sistmicos, 5) denervacin


vesical, 6) cistoplastias e aumento, y en ltimo trmino, 7) cistectoma.

Casos clnicos representativos

Un prosttico, sin otros problemas de salud, portador de sonda uretral pennanente,

4) TBe urogenital.

presenta bacleriuria

5)

(> 10'

unidades formadoras de colonias) en dos uroculti\los.

Cul es la actitud teraputica ms con\lenientel

Pielonefritis crnica por P. aeruginosa.

RC: 4

1 ) Tratamiento antibitico de amplio espectro.

2) Tratamiento antibitico segn antibiograma.

3) Cambio de sonda urinaria exclusivamente.


4) Antispticos en vejiga urinaria.
5) Cambio de sond..l urinaria y tratamiento antibitico.

derecha, fiebre de 39 "e, escalofros y sndrome micdonal acompaante. Es alrgica

RC: 3

1)

Ante un paciente de 24 aos, que presenta fiebre alta con dolor, inflamacin

Una paciente de 27 aos acude al servido de Urgencias por dolor en fosa renal
a penicilinas. Seale la respuesta correcta:

enrojecimiento testicular izquierdo, cul de las siguientes afirmaciones es INCO

2)
3)

RRECTA?

4)

1) El diagnstico ms probable es el de epididimitis.


2) los patgenos ms frecuentcs son Chlamydia lrachomalis y Neisseria gonorrhoeae.
3) El tratamiento de eleccin es vancomicna + gentamicina.
4) El tratamiento de eleccin puede ser ofloxacino.
5) Un tratamiento alternativo es ceftriaxona en monooosis ms 1 o das de doxiciclina.

5)

MIR 99-00, 135; RC: 3

No ser necesario descartar patologa urinaria obstrUd;va en este caso, ya que


presenta un claro sndrome miccional.
Para poder hacer el diagnstico de pielonefritis se deber conocer primero 10$
datos referidos a la funcin renal.
Se deber iniciar tratamiento emprico con un -Iactmico.
Si en las primeras horas evoluciona favorablemente podr continuar el tratamien
to de forma ambulante.
El mejor tratamiento disponible es la administracin intramuscular de aminoglu
csidos.

RC: 4

Un paciente de

83

aos sondado de forma permanente acude a la consulta tras

detectrsele dos cultivos positivos lomados con una semana de diferencia. Asegura
encontrarse asintomtico. la acttud ms adecuada ser:
Ante un paciente que presenta febrcula persistente, crisis renoureterales bre\les,
piuria estril, orina con pH cido, microhematuria persistente, con citologa uri
naria negati\la y epiddimos indurados .f.en qu enfermedad se debe pensar pri
mero?

1) Sarcoidasis.

2)

3)

Carcinoma vesical.
Carcinoma renal.

1)
2)
3)
4)
5)

Iniciar tratamiento antibitico segn antibiograma de los cuhivO$ obtenidos.


Tranquilizar al paciente y seguir con su pauta habitual de recambio de sonda.
Realizar cambio de sonda de forma inmediata con tratamiento antibitico.
Realizar cambio de sonda de forma inmediata con profila)(is antibitica de 4 das.

Retirar la sonda y colocar cistotoma suprapbica.

RC: 2

03.
UROLlTIASIS

OrientaCIn

MIR
Tema fundamental en
esta asignatura. Se debe
conocer muy bien la actitud
ante la litiasis en general,
ante los distintos tipos de
clculos y, especialmente,
todo lo relacionado con
el tratamiento. Es un tema
rentable y agradecido. as que
hay que emplear el tiempo
necesario. la tabla-resumen
de urolitiasis puede ser de
gran ayuda.

OJ

Los clculos ms frecuentes son los de oxalato clcico.

III

Globalmente, la litiasis es ms comn en el varn, salvo las de estruvita, ms comunes en mujeres.

La radiografa de abdomen no permite ver algunos clculos, como los de urato. Sin embargo, la ecografa
puede verlos, independientemente de su composicin.
Litiasis radiotransparentes: Sulfamidas, Indinavir, Urato, Xantinas (SIUX). las de cistina son radiolcldas; y
el resto, radioopacas.
las tiazidas son tiles para la hipercalciuria idioptica.
los clculos asociados a las resecciones ileales
clcico.

a la enfermedad inflamatoria intestinal son de oxalato

Precipitan en medio cido: cido rico y cistina. Precipitan en medio alcalino las que contienen fosfatos
(fosfato amnico magnsico o estruvita, y el fosfato clcico).
En el tratamiento de la litiasis por cido rico es beneficioso alcalinizar la orina.
los clculos de oxalato NO se ven alterados por el pH (al Oxal, el pH de la igual).
los clculos de estruvita se relacionan con microorganismos productores de ureasa, como Proteus.
las contraindicaciones absolutas para la LEOC son: embarazo, infeccin activa y obstruccin de las vas
urinarias distal al clculo.

3 . 1 . E p id e m iologa
Son numerosas las sustancias que se han identificado formando parte de 105 clculos. Su i ncidencia vara segn
el pas, e incluso segn las reas geogrficas dentro del mismo pas.
Se pueden distinguir seis grupos de componentes:
Oxalato clcico.
Fosfato clcico.
Fosfato no clcico.
Compuestos purnicos (cido rico, urato amnico, urato sdico, xantina, 2 , 8 dihidroxiadenina).
Aminocidos (cistina).
Otros (carbonato clcico, sulfamidas, etc.).

[?J

Preguntas

-MIR 08-09, 93
- MIR 06-07, 93, 106
-MIR 05-06, 104
-MIR 04-05, 104
- MIR 03-04, 44, 80
- MIR 02-03, 1 76
- MIR OO-01 , 1 1 7
- MIR 99-00, 1 89
- MIR 99-00F, 143, 146
- MIR 98-99, 135, 138
- MIR 98-99F, 144

Los clculos de oxalato clcico son los ms frecuentes, con cifras en torno al 65%, seguidos por los infectivos
y cido rico (alrededor del 15% cada uno). Fosfato clcico un 5%, y los de cistina con una incidencia baja
(1-3%).
La tercera dcada es la edad media de aparicin, por primera vez, de la litiasis salvo en los de cistina, que suelen
ser de aparicin ms prematura.
En Espaa, la incidencia de litiasis alcanza al 4,2% de la poblacin, con mayor afectacin de varones que mu
jeres. nicamente los clculos infectivos tienen mayor incidencia en la mujer.

03.
UROLlTIASIS

Aspectos esenciales

OrientaCin

MIR
TerT'kl fundamental en
esta asignatura. Se debe
conocer muy bien la actitud
ante la litiasis en general.
ante los distintos tijX)S de
clculos y. especialmente,
todo lo relacionado con
el tratamiento. Es un tema
rentable y agradecido, as que
hay que emplear el tiempo
necesario. La tabla-resumen
de urolitiasis puede ser de
gran ayuda.

los clculos ms frecuentes son los de oxalato clcico.


Globalmente, la litiasis es ms comn en el varn, salvo las de estruvita, ms comunes en mujeres.
la radiografa de abdomen no permite ver algunos clculos, como los de urato. Sin embargo, la ecografa
puede verlos, independientemente de su composicin.
Litiasis radiotransparentes: Sulfamidas, Indinavir, Urato, Xantinas (SIUX). las de cistina son radiolcidas; y
el resto, radioopacas.
las tiazidas son tiles para la hipercalciuria idioptica.
los clculos asociados a las resecciones ileales o a la enfermedad inflamatoria intestinal son de oxalato
clcico.

Precipitan en medio cido: cido rico y cistina. Precipitan en medio alcalino las que contienen fosfatos
(fosfato amnico magnsico O estruvita, y el fosfato clcico).
En el tratamiento de la litiasis por cido rico es beneficioso alcalinizar la orina.
los clculos de oxalato NO se ven alterados por el pH (al Oxal, el pH de la igual).
los clculos de estruvita se relacionan con microorganismos productores de ureasa, como Proteus.
Las contraindicaciones absolutas para la LEOC son: embarazo, infeccin activa y obstruccin de las vas
urinarias distal al clculo.

3 . 1 . E p i d e m iologa
Son numerosas las sustancias que se han identificado formando parte de los clculos. Su incidencia vara segn
el pas, e incluso segn las reas geogrficas dentro del mismo pas.
Se pueden distinguir seis grupos de componentes:
Oxalato clcico.
Fosfato clcico.
Fosfato no clcico.
Compuestos purnicos (cido rico, urato amnico, urato sdico, xantina, 2,8 dihidroxiadenina).
Aminocidos (cistina).
Otros (carbonato clcico, sulfamidas, ete.).

Preguntas

- MIR 08-09, 9 3
- MIR 06-07, lJ3. 106
- MIR 05-06, 104
- MIR 04-05, 104
- MIR 03-04, 44, 80
- MIR 02-OJ, ' 76

- MIR OO-Ol , 1 1 7
- MIR 99-00, 189
- MIR 99-00F, 143, 1 4 6
- MIR 98-99, 1 3 5 , 1 38
- MIR 98-99F, 144

Los clculos de oxalato clcico son los ms frecuentes, con cifras en torno al 65%, seguidos por los i nfectivos
y cido rico (alrededor del 1 5% cada uno). Fosfato clcico un 5%, y los de cistina con una i ncidencia baja
( 1 -3%).
La tercera dcada es la edad media de aparicin, por primera vez, de la litiasis salvo en los de cistina, que suelen
ser de aparicin ms prematura.
En Espaa, la incidencia de l itiasis alcanza al 4,2% de la poblacin, con mayor afectacin de varones que mu
jeres. nicamente los clculos infectivos tienen mayor incidencia en la mujer.

Manual eTO de Medicina y Ciruga, B.a edicin

o RECUERDA

coraliforme o "en asta de venado" [Figura 4]), manifestndose no como

las infecciones urinarias son ms frecuentes en mujeres que en varones.


Por eso los clculos de estruvita tambin lo son.

clico, sino como infecciones urinarias de repeticin, dolor lumbar


sordo, hematuria o incluso insuficiencia renal terminal.

La enfermedad litisica recidiva en el 40% de los casos, con una media


de un nuevo clculo cada dos o tres aos. Por recidiva se entiende la apa
ricin de una nueva litiasis de la misma composicin y en la misma lo
calizacin, en un intervalo menor de cuatro aos entre un clculo y otro.

3.2. M a n ifestaciones c l n i cas

y su manejo a g u d o
El dolor agudo del clico renal es l a manifestacin ms tpica de la li
tiasis renal. El dolor se produce por la sobredistensin de la va urinaria
tras la obstruccin de sta por el clculo. Es lgico, por tanto, que el
clculo deba desplazarse desde su origen calicial para producir sinto
matologa aguda. Ocasionalmente se observan cuadros de dolor vago
renal en relacin con litiasis caliciales no desplazadas.

Diagnstico

El clico renal o crisis renoureteral suele aparecer de forma progresi


va sobre la fosa lumbar afectada, irradindose por el flanco hacia la
ingle y los genitales (Figura 3). El paciente generalmente se encuentra

El anlisis bsico de orina muestra generalmente hematuria y leuco

afectado, con dolor que no cede con reposo, por lo que cambia de

cituria. Una piuria importante apoyara la posibil idad de infeccin

postura continuamente. Puede acompaarse de un cortejo vegetativo


con nuseas, vmitos y sudoracin. El dolor irradiado hacia la ingle

sobreaadida, aunque ninguno de estos datos es realmente determi


nante.

generalmente indica que el clculo ha alcanzado el urter. Cuando se


encuentra en vecindad de la vejiga, o bien dentro de sta, puede apare

Los cristales de oxalato clcico d i h idralado aparecen como bipir

cer un cuadro irritativo, similar al sndrome miccional con polaquiuria,

mides tetragonales a l observarlos con lupa b i nocular. Los de oxa


lato clcico monohidratado aparecen como cristales alargados que

disuria y tenesmo vesical.

adoptan forma de empalizada, formando clculos de estructura ra


diada, con aspecto compacto y macizo. Entre los fosfatos clcicos,
la brushita es el compuesto ms cido, formando cristales grandes
en forma de abanico de color azul con luz polarizada. Las apatitas

Sudoracin

tienen aspecto microgranular o esferoctico. El cido rico aparece


bajo la lupa como una desordenada aglomeracin de cristales. En
algunos clculos, los cristales estn tan juntos que se asemejan a u n a
masa continua.

RX simple de abdomen
con clculo a nivel de l3

La estruvita (o fosfato amnico magnsico) es el componente ms ca


racterstico de los clculos producidos por infeccin por grmenes
urealticos. Sus cristales tienen formas prismticas polimorfas, y rara
mente se observan los cristales "en atad" que pueden hal larse en el
sedimento. La cistina se reconoce fcilmente por su aspecto acarame
lado, formando cristales hexagonales en prismas o lminas.

Dolor de inicio

En teora, el 90% de los clculos son visibles en una radiografa simple


de abdomen, aunque este porcentaje es considerablemente menor en
Dolor irradiado

Sndrome miccional
Hematuria

las radiografas urgentes sin preparacin intestinal. Radiolgicamente,


la mayora de los clculos son radioopacos, exceptuando los de cido
rico y algunas otras composiciones infrecuentes (sulfamidas, xantina,
indinavir) (MIR 04-05, 1 04).

El estudio de imagen se completar mediante otras tcnicas diag


Figura 3. Diagnstico de urolitiasis

nsticas. La ecografa permitir visualizar incluso las l itiasis radio


transparentes, con el inconveniente de no ser vistas aquellas ubi
cadas en el trayecto ureteral (salvo las zonas cercanas a la vejiga o

Los clculos infectivos de estruvita, y en menor medida, los de cido

al rin). Tambin se podr evaluar el grado de hidronefrosis ( M I R

rico y cistina pueden crecer modelando las cavidades renales ( l itiasis

99-00, 1 89).

10

urolOgia
La urograffa ofrece informacin morfolgica y funcional de ambos
riones (Figura 5). Debe tenerse en cuenta que, durante el clico
renal, puede observarse una anulacin funcional, sin que signifi
que necesariamente deterioro de dicha unidad renal. Mediante
esta tcnica se puede diagnosticar todo tipo de clculos, ya sean
radiotransparentes o radioopacos. El principal inconveniente de
este procedimiento es la introduccin de contraste yodado, que est
contra indicado en los pacientes con alergia, creatin i na mayor de 2,
mieloma mltiple o deshidratacin i mportante.

Segn las guas clnicas, la urografa intravenosa (UIV) actualmente ha


sido desplazada por la Te helicoidal sin contraste, que se ha convertido
en el nuevo estudio de referencia para las litiasis. Aunque su alto coste
hace que todava no est extendido su uso, permite evaluar todo tipo
de clculos.

Figura 6. Doble J derecho. litiasis ureteral derecha. litiasis coraliforme izquierda

Tratamiento
El manejo agudo del clico renal se basa en el control del dolor.
Para esto, es preciso conseguir una disminucin de la presin den

3.3. Eva l u acin y trata m iento


de l a l itiasis ren a l

tro de la va urinaria, Jo que puede hacerse, sobre todo, con anti


inflamatorios, que disminuyen el dolor y la d iuresis a l i n hibir la
sntesis de prostaglandinas. Asimismo, se pueden usar espasmolti
cos, que disminuyen la presin intraureteral al relajar la pared del
urter.

Este apartado se puede dividir en dos partes. Por u n lado, el estudio


de la litiasis con la finalidad de instaurar un tratamiento preventivo
de su formacin, y por otro, el estudio y tratamiento de la litiasis ya
formada.

Existen una serie de situaciones en las que el clico renal se convierte

en una urgencia que precisa de hospitalizacin, y eventualmente, de


manipulacin invasiva:
Obstruccin grave, principalmente si se acompaa de litiasis mayor
de 1 0 mm.

Estudio
y tratamiento preventivo

Fiebre elevada (mayor de 38 OC).


Dolor incontrolable.
Rin nico.
Asimismo, en pacientes diabticos, por el mayor riesgo de com

La evaluacin del paciente con litiasis se basa en un estudio meta


blico para determinar qu factores son modificables, en un intento
de evitar la recidiva (Tabla 5).

pl icaciones, es aconsejable, si no el i ngreso, a l menos una obser


vacin estricta. Una situacin s i m i l a r ocurre durante el emba razo,
donde una dilatacin leve de la va urinaria puede considerarse

Este estudio debe reservarse para aque


l l os pacientes con alta proba b i l idad de

" fisiolgica", pero obstrucciones ms importantes o la aparicin

recidiva, aunque cada vez ms autores

de fiebre hacen aconsejable l a colocacin de u n catter u reteral

indican que debe realizarse a todos Jos


pacientes (Tabla 6).

(Figura 6).

a RECUERDA

la furosemida, al re
vs que las tiazidas,

aumenta el calcio uri


nario.

11

Manual CTO de Medicina y Ciruga, B.a edicin

LITIASIS CALCICA
(OXALATO O FOSFATO)

LITIASIS
URICA

(ESTRUVITA)

Hipercalciuria idioptica

Gota primaria

Hipercalciuria secundaria

Hemopatas

por grmenes

a hipercalcemia

Enfermedades

productores

digestivas

de ureasa

Hiperuricosuria

Hiperoxaluria

Hipocitraturia

plante heptico, que suele ir unido al renal, aunque algunos casos


responden a piridoxina.

LITIASIS
INFECCIOSA

LITIASIS
CtSTINICA
. Cistinuria

. Infecciones

11 RECUERDA

La causa ms frecuente de hipercalcemia en un pac iente ambulatorio


es el hiperparatiroidismo primario. En cambio, la hipercalcemia ms
frecuente en uno ingresado es la de origen neoplsico.

Ingesta excesiva
de purinas

Acidosis renal tubular

Frmacos

distal

litiasis rica

litiasis clcica idioptica

No obstante, la mayora de los casos de hiperoxaluria son secun


darios a malabsorcin de cidos grasos por enfermedades crnicas

idioptica

pancreatobiliares, derivacin intestinal para el tratamiento de la

Tabla 5. Tipos de litiasis. Situaciones que favorecen su aparicin

obesidad mrbida, reseccin ileal, enfermedad inflamatoria intes


tinal (MIR 05-06, 1 04; MIR 00-0 1 , 1 1 7), hipercalciuria coincidente

Edad temprana de aparicin

o por falta de calcio en la dieta, lo que permite que exista mayor

Litiasis bilateral

cantidad de oxalato intestinal para su absorcin.

litiasis en rin nico o malformado

La intoxicacin por etilenglicol y metoxiflurano puede producir hi

Composicin poco frecuente

peroxaluria, as como la ingesta de vitamina C en altas dosis. En

litiasis recidivante

todos estos casos secundarios, el tratamiento con colestiramina, una


dieta pobre en grasas y la correccin de la malabsorcin, en la me

Nefrocalcinosis
Litiasis coraliforme

dida de lo posible, suelen ser medidas eficaces.

Tabla 6. Pacientes con indicacin de estudio metablico

Hipocitraturia. Excrecin de citrato inferior a 300 mgl24 h. Gene

ralmente se asocia a otras anomalas urinarias. Aunque de causa


Desde el punto de vista prctico, las litiasis se pueden dividir en: las

desconocida, puede contribuir una dieta rica en protenas, hipoca

de composicin clcica y las de otras composiciones, ya que el primer


grupo supone la mayora de los casos (70-80%) tratados habitualmente.

Hiperparatiroidismo primario. Supone la causa ms frecuente de

liemia, enfermedad intestinal o infeccin urinaria.


hipercalciuria conocida (vase Seccin de Endocrinologa, metabo
lismo y nutricin) (MIR 98-99F, 1 44).

Acidosis tubular renal distal (vase Seccin de Nefrologa). Enfer

Litiasis clcica

En la mayora de las ocasiones se desconoce el origen de la litiasis cl

medad autosmica recesiva. Consiste en la imposibilidad del tbulo


distal para excretar hidrogeniones a la orina (orinas persistentemen

cica, aunque se puede hacer una aproximacin a los factores de riesgo

te alcalinas) con aumento de la eliminacin de calcio a la orina.

que influyen en su aparicin. Slo en un pequeo porcentaje de casos


existe una enfermedad de base que puede ser tratada, y de esta forma

res de clculos de oxalato clcico y con hipercalciuria idioptica.

Existen formas incompletas que se observan en pacientes formado


En stos, probablemente la acidosis tubular no juegue un papel im

desaparece la formacin de clculos clcicos.


Hipercalciuria idioptica. Es la causa ms frecuente de litiasis cl

portante y responden a tiazidas (MIR 98-99, 1 38).

cica. Se define como una excrecin urinaria de calcio mayor de

Otras circunstancias que favorecen la litiasis clcica son: sarcoido

300 mgl24 h en el varn y 250 mgl24 h en la mujer. De cara a su


manejo, las tiazidas disminuyen el calcio urinario, reduciendo la

anomalas en el pH urinario (alcalosis).

sis, sndrome de Cushing, diuresis escasa, dficit de inhibidores o

formacin de litiasis (MIR 03-04, 44; MIR 99-00F, 1 46; MIR 98-

Litiasis clcica idioptica. Aproximadamente en el 20% de los pa

99, 1 35). La administracin de citrato potsico ayuda a evitar la

cientes con litiasis clcica no se demuestra ninguna anomala en el

hipopotasemia y aumenta el citrato urinario, que es inhibidor de la

estudio metablico.

litognesis (Tabla 7).


I

ABSORTIVAS

RESORTIVAS

RENALES

Aporte excesivo

Hiperparatiroidismo

Acidosis tubular

Sd. de Surnen (leche

Inmovilizacin

distal

Tumorales

Idioptca

Sd. de Cushing

y alcalinos )

Hipervitamlnosls O
Idioptica

Enf. Paget

Sarcoidosis

El cido rico no disociado es poco soluble en orina. Con un pH uri


nario de 5, la solubilidad del cido rico es nicamente de 1 00 mgll,
mientras que con un pH de 7 es de 1 .580 mgll. Esto demuestra la gran
importancia del pH urinario en la formacin de clculos de cido ri
co. Aparte de stos, tambin existe una pequea proporcin de clcu

Tabla 7. Causas ms frecuentes de hipercakiuria

12

Litiasis rica

los de urato monosdico y urato amnico.

Hiperuricosuria. Excrecin en orina de ms d e 800 mgl24 h en el

El objetivo del tratamiento es reducir el cido rico excretado y au

varn o 750 mgl24 h en la mujer. Adems de favorecer la litiasis

mentar el pH urinario (MI R 03-04, 80), ya que los clculos ms fre

rica, la hiperuricosuria constituye un factor de riesgo para la for

cuentes en pacientes hiperuricmicos son los de cido rico. Por otra

macin de clculos de calcio, probablemente por nucJeacin hete


rognea sobre ncleos de cido rico o urato sdico. Generalmente

to mdico mediante quimilisis por alcalin izacin urinaria. Pueden

se debe a un exceso de purinas en la dieta.

administrarse diversos lcalis; el citrato potsico impedira el teri

Hiperoxaluria. Se considera como tal la excrecin en orina de ms

co riesgo de formacin de clculos clcicos por su efecto inhi bidor,

parle, este tipo de clculos son los que mejor responden al tratamien

de 40 mgl24 h. Existe una hiperoxaluria primaria, que es conse

pero tambin pueden tratarse con bicarbonato sdico o citrato sdi

cuencia de un defecto enzimtico autosmico recesivo; no tiene

CO

tratamienlo y generalmente conduce a insuficiencia renal por litiasis

250 mglda. Cuando, adems, la uricemia es alta, puede tratarse con

recidivante. El nico tratamiento que existe actualmente es el tras-

alopurinol (MIR 06-07, 93).

(MIR 02-03, 1 76). Una a lternativa es la acetazolamida en dosis de

urolOgia

Litiasis cistnica

complejo enzima-inhibidor irreversible. Se utilizan bsicamente dos


sustancias de esta naturaleza: el cido propinico y el acetohidroxmi

La cistinuria es un trastorno autosmico recesivo en el que existe un


defecto de absorcin, a nivel intestinal y tubular proximal, de los ami

co. Su empleo suele venir acompaado de cefaleas, temblores, trom


bosis venosas u otros sntomas neurolgicos, por lo que tampoco son

nocidos dibsicos: cistina, ornitina, lisina y arginina (COLA), aunque

de gran aceptacin.

parece que puede existir un trastorno en el que nicamente se ve afec


tada la cistina, lo que i ndicara que, adems de un mecanismo de trans
porte comn, existe uno independiente para la cistina.
Los niveles de cistina en orina de 24 horas son superiores a 1 00 mg, de
hecho, los homoeigotos pueden excretar ms de 600 mglda. El diag

Todo lo relativo al estudio de la nefrolitiasis expuesto anteriormente se


puede repasar en la Tabla 8.

Tratamiento de la litiasis ya formada

(Figura 7)

nstico se realiza identificando los caractersticos cristales hexagonales


en orina, o por una prueba positiva de nitroprusiato sdico (la orina se
tie de azul en pacientes afectados por esta enfermedad: test de Brand).

Los clculos ya formados no expulsables (> 4-5 mm) precisan de

El tratamiento consiste en aumentar la diuresis diaria (ms de 3 I/da),

tratamiento "agresivo", es decir, necesitan ser extrados qui rrgica


mente o fragmentados de forma que puedan ser expu lsados espon

alcalinizar la orina por encima de 7,5 y, en caso de que esto sea insufi
ciente, puede iniciarse tratamiento con D-penicilamina (250 mgl6 h) o

tneamente.

a-mereaptopropionilglieina (250 mgl6 h).

A continuacin se analizan brevemente las diversas formas de tratamiento.


Ciruga.

Ha

sido el tratamiento estndar hasta la aparicin de la

litotricia extracorprea. An hoy da, es preciso recurrir a la ciruga


cuando fracasan las ondas de choque o en determinados casos para

Litiasis infectiva

reducir la masa litisica (clculos coraliformes).


Los clculos infectivos de estruvita o de fosfato amnico magnsico (Mg

NHl04-6H20) se desarrollan

en un ambiente alcalino, producido por

Endourologa. La manipulacin endoscpica de la va urinaria es


cada da ms accesible gracias a las mejoras tcnicas. Puede reali

infeccin persistente de grmenes que hidrolizan la urea, aumentando

zarse extraccin directa del clculo mediante diversos tipos de pin

la cantidad de amonio urinario. Los principales grmenes que poseen

zas o cestillas, o bien fragmentar previamente el clculo mediante

ureasa, adems de diversas especies de Proteus (MIR 06-07, 1 06). son

diversas fuentes de energa, como la electrohidrulica, ultrasnica o

Pseudomonas, Klebsiella, Serratia y Enlerobacter. La presencia de cuer

lser. Se puede acceder hasta el clculo mediante ureterorrenosco

pos extraos (sondas vesicales, suturas) favorece su formacin.

pia (U RS) o nefrolitotoma pereutnea ( NLPC).


litotricia extracorprea por ondas de choque (LEOC). Las ondas de

Para su tratamiento se han empleado diversos mtodos, generalmente


ineficaces. La antibioterapia nicamente mantiene estril la orina du
rante los cursos de tratamiento.

choque se transmiten a travs de los tej idos corporales con la misma


impedancia acstica que el agua hasta alcanzar la litiasis, sobre la
que produce fenmenos de compresin y descompresin que con
ducirn a su fragmentacin.

Parece ms prometedor el uso de inhibidores de la ureasa con cidos

Prcticamente todos los clculos son susceptibles de tratamiento me

hidroxmicos. Estos son molculas anlogas a la urea que forman un

diante LEOe. La nica limitacin seran aquellos clculos no localiza-

SALES CALCICAS
Frecuencia
Sexo

. Oxalato clcico: 55-60%

AClDO URICO

10-15%

5-10%

1 -3%

Varn

Mujer

Varn

Varn = Mujer

Infeccin por grmenes ureasa (+)

Gota (50%)
. Idioptica 50%)
. Hiperuricemias secundarias

Cistinuria

cido

Idioptica
Hiperuricosuria (20%)

pH

Alcalino

Alcalino

cido

Radiologia

Radioopacos

Radioopacos

Radiotransparentes

:liD

Moologa
de los cristales

Cristales de fostato Ca

1Iil' -ft>

D fSJ

Cristales de OxCa

Tratamiento

CISTlNA

Fosfato clcico: 1 0-' 5%

Hipercalciuria idioptica
Etiologia

ESTRUVITA (FOSFATO
AMNICO MAGNSICO)

Hipercalciuria idioptica: tiazidas


Hiperoxaluria 1 .11; piridoxina
Hiperoxaluria 2.11; colestiramina

Formas prismticas polim6rficas


Cristales-en atad-

q
1lJ

Cristales de estruvita

cido propi6nico y cido


acetohidroxmico
Antibioterapia
En ocasiones ciruga

Aglomerados de cristales
desorganizados, a veces formando
masas continuas

&o 0
O rfF
Cristales de cido rico
Alcalinizar la orina
Alopurinol (si hay hiperuricemia)
Dieta de bajo contenido
proteico

Rad iolcidos

Cristales hexagonales en prismas


o lminas

W
OO
Cristales de cistina
Forzar diuresis (ingesta hdrical
Alcalinizar orina
D-penicilamina (si no hay
respuesta)

Tabla 8. Tabla-resumen de las nefrolitiasis

13

Manual CTO de Medicina y Ciruga, s.a edicin


de su tamao, composicin y
dureza, localizacin, particula

MANEJO DE LA UROLlTIASIS

Crisis
CUADRO AGUDO

TTO. SINTOMATICO
Espasmolticos
y antiinflamatorios
Reposo e hidratacin

cretora y paciente, funcin renal


y tipo de litotriptor disponible.

Estable

CUADRO CRNICO

No
comlicado

ridades anatmicas de la va ex

-Complicado

Obstruccin gr ave
Infeccin, fiebre

Dolor incoercible
Rin nico

Tratar la condicin
preexistente

Ecografa renal

Dilatacin
INGRESO YTTO. AGRESIVO
DRENAJE
(catter o nefrostoma)
no. PARENTERAL
Antibiticos
Remontar hemodinmica
Equilibrio electroltico
Narcticos
VIGILANCIA ESTRECHA

CALCICA
acidificar
(no til si oxalato),
citratos, tiazidas
colestiramina

hiperoxaluria
RICA
alcalinizar,
alopu rino l
ESTRUVITA
acetohid rox m ico
CISTINA
O-penicilamina,
vit. B y alcalinizar

deber ser estabil izada previa


mente a la misma y constituye,
en c ierto modo, por ello, con

traindicacin relativa de LEOC

URETEROTOMIA
PIELOllTOTOMIA
NEFRECTOMIA

CONTRAINDICADA EN
EMBARAZO

el riesgo de hemorragia duran


te la sesin de litotricia, luego

Ciruga

EXTRACORPREA
(lEOCJ
PERCUTANEA
ultrasonogrfica
con microlumbotoma
ENOOSCPICA
va ureteral

y dieta baja en
grasas y rica
en calcio, si

arterial no controlada facilita

litotricia

La presencia de hipertensin

Edad?
Tipo de clculos?
Periodicidad de la clnica?
Tipo de sntomas?
Viabilidad renal?

Indican o
no LEOC o
ciruga

INFECCIN
OBSTRUCCIN DISTAL
Aneurismas
Coagulopatfas
Obesidad
Arritmia cardaca

(MIR 08-09, 93; M I R 99-00F,


1 43 ) (Tabla 9).

Complicaciones

La expulsin de fragmentos Ii
tisicos puede ocasionar un
clico renal y, con menor fre
cuencia, obstruccin ureteral
(sleinstrasse o "calle litisica/l).

Esta posib i l idad es mayor ante


litiasis de gran tamao, por lo
que en algunos de estos casos
se puede colocar un catter
de derivacin urinaria (ne

Figura 7. Manejo de la litiasis renal ya formada

frostoma o doble J) antes de


bIes por su pequeo tamao 2-5 mm). Cualquier litiasis podra ser

la LEOC para d i s m i n u i r este riesgo, generalmente en litiasis supe

tratada con LEOC, aunque esto tendr que ser matizado en funcin

riores a 2 c m .

ABSOLUTAS

Embarazo
Obstruccin distal
Infeccin activa

RElATIVAS (PRECISAN DE CONTROL PREVIO

Derivadas del efecto directo de las ondas de choque, pueden aparecer

AL TRATAMIENTO)

contusiones renales manifestadas como hematuria, hematomas rena

Alteraciones de la coagulacin

Aneurisma artico
Alteraciones del ritmo cardaco, marcapasos
o desfibriladores
Obesidad
Hipertensin arterial descontrolada

Tabla 9. Contraindicaciones de la LEOC (MIR 99-00F, 143)

hematuria se considera la complicacin ms frecuente de la litotricia.


Ms controvertida es la terica relacin de la LEOC con la aparicin de
hipertensin arterial, ya que no est demostrada en las ltimas revisiones
publ icadas, aunque s la relacin entre hematoma renal post-LEOC e
hipertensin arterial.

A un hombre de 29 aos, con antecedentes de dolor tipo clico en fosa renal izquierda
que cedi con tratamiento analgsico, se le practica una urografa intravenosa, apre
cindose defecto de replecin radiotransparente de 6x7 mm en tercio distal de urter
izquierdo. El pH de la orina fue de 5,5; asimismo, se observan cristales de urato, 9-12
hemates por campo y escasa leucocituria. Cul sera el tratamiento ms apropiadol

das de evolucin, asociado en las ltimas 24 horas a fiebre, escalofros y malestar


general. Analtica de sangre: plaquetopenia, leucocitosis y disminucin de la activi
dad de la protrombina. Analtica de orina normal. Radiografa de abdomen con claras
imgenes de litiasis. Eco renal: dilatacin moderada de sistema excretor izquierdo.
Cul es la conducta ms adecuada?

1 ) Alopurinol va oral.

1 ) Solicitar hemocultivos y urocultivo para establecer la necesidad de antibiote

2) Ureteroscopia con extraccin del clculo.


3) Nefrolitotoma endosc6pica percutnea.
4) Alcalinizacin de la orina por va oral.
5)

Administracin de D-penicilamina.

MIR 03-04, 80; RC: 4


Mujer de 50 aos, diabtica insulinodependiente, con infecciones urinarias y clicos
nefrticos de repeticin. Acude a Urgencias por dolor en fosa renal izquierda de cinco

14

les, equimosis o eritema cutneo, y en grado mximo, rotura renal. La

ra p i a.
Realizar urografa intravenosa para intentar filiar la causa.
3) Hidratar a la paciente bajo observacin rigurosa, y repetir ecografa a l as 48
horas.
4) Colocar catter doble) o practicar nefrostoma percutnea de forma inmediata
con cobertura antibitica.
5) laparotoma exploradora para objetivar causa, y realizar tratamiento antes de que
el cuadro est muy evolucionado.
2)

RC: 4

04.
TUMORES RENALES

Aspectos esenciales

OnentaCln

MIR
El aclenocarcinoma

fenal es un

tema "de moda" en el examen

MIR. Cualquier aspecto de


este lema puede aparecer,
pero es fundamental reconocer
el sndrome de $Iauffer,
que ha sido recientemente
introducido y preguntado

varias veces consecutivas.

(jJ

El ms frecuente de los tumores slidos renales es el hipernefroma.

lIJ

El paciente caracterstico es un varn de mediana edad, obeso y fumador.

GJ

la trada clsica consiste en hematuria, dolor y masa en flanco, actualmente, lo ms habitual es que sea
incidentaloma (asintomtico). Si produce sntomas, el ms frecuente es la hematuria.
Hay que sospechar tumor renal ante un varicocele izquierdo, de aparicin sbita y que no cede con el
decbito.
El hipernefroma puede producir multitud de sndromes paraneoplsicos. Esto puede complicar bastante el
diagnstico, de ah el sobrenombre de "tumor del internista".
La elevacin de las transaminasas sin afectacin heptica es tpica del hipernefroma (sndrome de Stauffer).
No se debe confundir un quiste simple con un hipernefroma. los criterios de quiste simple son: contorno
liso, contenido transnico y refuerzo posterior.
la primera prueba de imagen, ante la sospecha de hipernefroma, sera la ecografa.

El tratamiento fundamental del hipernefroma es la extirpacin quirrgica. La quimioterapia y radioterapia


tienen un papel muy secundario.

4.1 . Ca rc inoma de c l u las rena les


(adenoca rc inoma re nal, h ipernefroma)
Es e l tumor slido renal ms frecuente (90%) (Figura 8) (MIR 99-00, 1 77). Es un tumor fundamentalmente d e l a
edad adulta, con mayor incidencia entre los 4 0 y 60 aos, con predominio e n e l varn 2 : 1 a excepcin d e la
variedad cromfoba, tpica de las mujeres.
Entre los factores de riesgo que se han implicado se encuentran el humo del tabaco, el cadmio y la obesidad.
Existen formas familiares que suelen ser mltiples y bilaterales, como en la enfermedad de Von Hippellindau y,
en menor medida, la esclerosis tuberosa (MIR 0001 , 1 20). Se han identificado a lteraciones cromosmicas que
implican al cromosoma 3.

D RECUERDA
IIJ

Preguntas

MIR 09-10, 99
- MIR 06-07, 102
- MIR 05-06, 105
- MIR 04-05, 105
- MIR 01-02, 109
- MIR 00-0 1 , ' 2 0
- M I R 99-00, 1 7 7
- MIR 99-00F, 144
- MIR 98-99F, 146

la esclerosis tuberosa y la enferme


dad Von Hippel.lindau se asocian
tambin a otro tumor renal, el an
giomiolipoma.

Asimismo, existe una incidencia aumentada en el rin poliqustico,


en la enfermedad qustica adquirida de la insuficiencia renal crnica
y en los riones malformados, como el rin "en herradura". Procede
de las clulas del tbulo contorneado proximal, y microscpica mente
predominan las clulas claras sobre las granulares y fusiformes .

Presentacin
La trada clsica: hematuria dolor y masa en el flanco ocurre nicamente en el 1 0% de los casos y, cuando se
presenta as, generalmente se trata de una enfermedad avanzada.

15

Manual eTO de Medicina y Ciruga, B.a edicin


de su tamao, composicin y
dureza, localizacin, particula

MANEJO DE LA UROllTIASIS

ridades anatmicas de la va ex
cretora y paciente, funcin renal

Crisis

Estable

CUADRO AGUDO

CUADRO CRNICO

/
t

nO. SINTOMATICO

Espasmolfticos
y a ntiinflamatorios
Reposo e hidratacin

Obstruccin grave
. Infeccin, fiebre

Dolor incoercible
Rin nico

Tratar la cond icin


p reexistente

Ecografa renal

Dilatacin
INGRESO VITO. AGRESIVO
DRENAJE

(catter o nefrostoma)
no. PARENTERAL
Antibiticos
Remontar hemodinmica
Equi librio electroltico
Narcticos
VIGILANCIA ESTRECHA

traindicacin relativa de LEOC

acidificar
(no til si oxalato),
citratos, tiazidas
colestiramina
y dieta baja en
grasas y rica
en calcio, si
hi peroxal uria

(LEOCI

ESTRUVITA

acetohidroxmico
CISTlNA
D-penici lam i na,

vit. B6 y alcalinizar

te la sesin de litotricia, luego


deber ser estabil izada previa
mente a la misma y constituye,
en cierto modo, por ello, con

EXTRACORPREA

RICA

el riesgo de hemorragia duran

Ciruga

CALCICA

alcali nizar,
alopurinol

arterial no controlada facilita

litotricia

La presencia de h ipertensin

Edad?
Tipo de clculos?
Periodicidad de la clnica?
Tipo de slntomas?
Viabilidad renal?

Indican o
no LEO( o
ciruga

Compl icado

y tipo de l i lotriptor disponible.

URETEROTOMIA
PIELOLITOTOMIA
NEfRECTOMIA

PERCUTANEA

ultrasonogrfica
con microlumbotoma
va ureteral

La expulsin de fragmentos Ii
tisicos puede ocasionar un

CONTRAINDICADA EN

EMBARAZO

clico renal y, con menor fre


cuencia, obstruccin ureteral

INFECCIN
OBSTRUCCIN DISTAL
Aneurismas
Coagulopatfas
Obesidad
Arritmia cardiaca

(steinstrasse o "calle litisica").

Esta posibilidad es mayor ante


litiasis de gran tamao, por lo
que en algunos de estos casos
se puede colocar un catter

Fig ura 7. Manejo de la litiasis renal ya formada

de derivacin urinaria (ne


frosloma o doble J) antes de

bies por su pequeo tamao 2-5 mm). Cualquier litiasis podra ser

la LEOC para dismi n u i r este riesgo, generalmente en litiasis supe

tratada con LEOC, aunque esto tendr que ser matizado en funcin

riores a 2 cm.

ABSOLUTAS

Embarazo
Obstruccin distal
I nfeccin activa

RelATIVAS (PRECISAN DE CONTROL PREVIO

Derivadas del efecto directo de las ondas de choque, pueden aparecer

Al TRATAMIENTO)

contusiones renales manifestadas como hematuria, hematomas rena

Alteraciones de la coaguladn
Aneurisma artico
Alteraciones del ritmo cardiaco, marca pasos
desfibriladores
Obesidad
Hipertensin arterial descontrolada

Tabla 9. Contraindicaciones de la LEOC (MIR 99-OOF, 143)

les, equimosis

eritema cutneo, y en grado mximo, rotura renal. La

hematuria se considera la complicacin ms frecuente de la litotricia.


Ms controvertida es la terica relacin de la LEOC con la aparicin de
hipertensin arterial, ya que no est demostrada en las ltimas revisiones
publicadas, aunque s la relacin entre hematoma renal post-LEOC e
hipertensin arterial.

A un hombre de 29 aos, con antecedentes de dolor tipo clico en fosa renal izquierda
que cedi con tratamiento analgsico, se le practica una urografa intravenosa, apre.
cindose defecto de replecin radiotransparenle de 6x7 mm en tercio distal de urter
izquierdo. El pH de la orina fue de 5,5; asimismo, se observan cristales de urato, 9-12
hemates por campo y escasa leucocituria. Cul sera el tratamiento ms apropiado?

das de evolucin, asociado en las llimas 24 horas a fiebre, escalofros y malestar


general. Analtica de sangre: plaquetopenia, leucocitosis y disminucin de la activi
dad de la protrombina. Analtica de orina normal. Radiografia de abdomen con daras
imgenes de litiasis. Eco renal: dilatacin moderada de sistema excretor izquierdo.
Cul es la conducta ms adecuadal

1 ) Al opurinol va ora l.
2) Ureleroscopia con exlraccin del clculo.

1 ) Solicitar hemocu lti vos y urocultivo para cstablecer la necesidad de antibiotc

3)

2) Realizar urografa intravenosa para intentar filiar la causa.


3) Hidratar a la paciente baj o observacin rigurosa, y repetir ecografa a las 48
horas.
4) Colocar catter doble J o practicar nefrostomfa percutnca de forma inmediata

Ncfrolitoloma endoscpica percutnea.


4) A lca lini zlcin de la orina por va oral .
5) Admin ist racin de D-penicilamina.
MIR 0304, 80; RC: 4

Mujer de 50 aos, diabtica insulinodependiente, con infecciones urinarias y clicos


nefrticos de repeticin. Acude a Urgencias por dolor en fosa renal izquierda de cinco

14

1 43) (Tabla 9).

Complicaciones

ENOOSCPICA

(MIR 08-09, 93; M I R 99-00F,

rapia.

con cobertura antibitica.

5) laparotoma exploradora pJra objetivar causa, y re;lizar tratamiento antes de que


el cuadro est muy evolucionado.

RC: 4

04.
TUMORES RENALES

Aspectos esenciales

Orll?ntacln

MIR
El adenocarcinoma renal es un
tema "de moda en el examen
MIR. Cualquier aspecto de
este lema puede aparecer,
pero es fundamental reconocer
el sindrome de Stauffer,
que ha sido recientemente
introducido y preguntado
varias veces consecutivas.

El ms frecuente de los tumores slidos renales es el hipernefroma.


El paciente caracterstico es un varn de mediana edad, obeso y fumador.
la trada clsica consiste en hematuria, dolor y masa en flanco, actualmente, lo ms habitual es que sea
incidentaloma (asintomtico). Si produce sntomas, el ms frecuente es la hematuria.
Hay que sospechar tumor renal ante un varicocele izquierdo, de aparicin sbita y que no cede con el
decbito.
El hipernefroma puede producir multitud de sndromes paraneoplsicos. Esto puede complicar bastante el
diagnstico, de ah el sobrenombre de "tumor del internista".
la elevacin de las transaminasas sin afectacin heptica es tpica del hipernefroma (sndrome de Stauffer).
No se debe confundir un quiste simple con un hipernefroma. los criterios de quiste simple son: contorno
liso, contenido transnico y refuerzo posterior.
la primera prueba de imagen, ante la sospecha de hipernefroma, sera la ecografa.
El tratamiento fundamental del hipernefroma es la extirpacin quirrgica. la quimioterapia y radioterapia
tienen un papel muy secundario.

4.1 . Ca rci noma de c l u l a s ren a l es


(adenocarc i n oma ren a l, h i pernefroma)
Es e l tumor slido renal ms frecuente (90%) (Figura 8) (MIR 99-00, 1 77). Es un tumor fundamentalmente d e l a
edad adulta, con mayor incidencia entre los 40 y 60 aos, con predominio e n e l varn 2 : 1 a excepcin d e la
variedad cromfoba, t pi ca de las mujeres.
Entre los factores de riesgo que se han impl icado se encuentran el humo del tabaco, el cadmio y la obesidad.
Existen formas familiares que suelen ser mltiples y bilaterales, como en la enfermedad de Von

Hippel -Li ndau y,

en menor medida, la esclerosis tuberosa (MIR 00-01 , 1 20). Se han identificado alteraciones cromosmicas que
implican al cromosoma 3 .

D RECUERDA
III

Preguntas

la esclerosis tuberosa y la enferme


dad Von Hippel-lindau se asocian
tambin a otro tumor renal, el an
giomiolipoma.

Asimismo, existe una incidencia aumentada en el rin poliqustico,


en la enfermedad qustica adquirida de la insuficiencia renal crnica
y en los riones malformados, como el rin "en herradura". Procede
de las clulas del tbulo contorneado proximal, y microscpicamente
predominan las clulas claras sobre las granulares y fusiformes .

MIR 09-1 0, 99

- MIR 06-07, 102


. MIR 0506, 105
. MIR 04-05, 105
- MIR 01-02, 109
- MIR OO-Ol, 1 20
- MIR 99-00, 177
- MIR 99-00F, 144
- MIR 98-99F, 146

Presentacin
La trada clsica: hematuria, dolor y masa en el flanco ocurre nicamente en el 1 0% de los casos y, cuando se
presenta as, generalmente se trata de una enfermedad avanzada.

15

Manual (TO de Medicina y Ciruga, 8.a edicin

Figura 9. Ecografa de quistes renales simples

la realizacin de puncin-aspiracin con aguja fina (PAAF) de una


Figura 8. Carcinoma de clulas renales

masa renal para su filiacin es una exploracin agresiva que, debido


a su baja sensibilidad, no se justifica actualmente, excepto en casos
excepcionales.

El 30% presenta metstasis a distancia en el momento del diagnstico,


realizar ecografas abdominales de rutina por otra causa, alcanzando

la tomografa axial computarizada (fe) es el mejor mtodo aislado para


evaluar una masa renal, proporcionando informacin precisa sobre mets

en algunos estudios ms de la mitad de los casos diagnosticados.

tasis ganglionares (80%) y afectacin de rganos adyacentes (Figura 1 0).

aunque contrariamente, cada vez son ms los hallazgos incidentales al

la anomala ms frecuente es la hematuria macroscpica o micros


cpica (60%). Otros hallazgos frecuentes son dolor (40%), prdida de
peso (30%), anemia (40%), masa en flanco (24%), HTA (20%), hiper
calcemia (6%), eritrocitosis (3%). El 20% de los pacientes presentan
como cuadro paraneoplsico alteracin de las enzimas hepticas sin
evidencia de metstasis (sndrome de Stauffer) (MIR 09- 1 0, 99; MIR 0607, 1 02 ; M I R 05-06, 1 05).
Ocasionalmente, el adenocarcinoma renal puede producir hormonas
productoras de sndromes clnicos segn la sustancia secretada. Entre
stas se encuentran pptidos PTH-like, prostaglandinas, prolactina, re
nina, gonadotropinas o corticoides. la invasin de la vena renal princi
pal en el lado izquierdo puede ocasionar la aparicin de un varicocele
de forma repentina, que no disminuye en decbito.

o RECUERDA

la produccin de pptidos puede aparecer en el hipernefroma, pero es


ms tpica de carcinomas epidermoides (pulmn, esfago, etc.).

"

"

Diag nstico

.=
La ecografa es la primera prueba complementaria que debe realizarse,

La RM, aunque no se emplea de manera rutinaria en este tipo de pa

de forma que la identificacin ecogrfica de una lesin que cumple cri


terios de quiste simple (contorno liso, contenido transnico y refuerzo

cientes, s se utiliza como mtodo de diagnstico bsico en sujetos en

posterior) hace innecesarios mayores esfuerzos diagnsticos, pudiendo


hacerse un seguimiento ecogrfico anual (MIR 99-00F, 1 44). De esta

cava (MIR 04-05, 1 05) (Figura 1 1 ).

forma se diagnostican l a mayora de las masas renales en la actualidad

Aunque la urografa intravenosa (U IV) contina siendo la base del diag

(Figura 9).

nstico por imagen en urologa, en el caso del adenocarcinoma renal

16

los que se sospecha afectacin lrombtica tumoral de la vena renal o

UrOIOga

proporciona pocos datos y de forma indirecta, como puede ser la dis


torsin del sistema colector, su ocupacin o la anulacin funcional del
rin. En las placas tomogrficas de la UIV puede observarse la presen
cia de una masa o una alteracin del contorno renal.

Trata miento
Una vez estudiado el tumor y descartada la presencia de metstasis,
tanto viscerales como ganglionares, el tratamiento de eleccin es la
nefrectoma radical, incluyendo la fascia de Gerota y la glndula su
prarrenal (MIR 01 -02, 1 09).
En ocasiones especiales se puede plantear la ciruga conservadora o
parcial, como en los tumores bilaterales, en aquellos que aparecen so
bre rin nico, o sujetos con nefropatas mdicas, en los que la prdi
da de masa nefronal obligara a dilisis.
Adems de en estas indicaciones imperativas, actualmente es el nuevo
patrn de referencia en el tratamiento de tumores pequeos (menos de
4 cm), bien delimitados y sin afectacin de la grasa perirrenal. En estos
pacientes seleccionados parece que la supervivencia y la tasa de reci
divas locales son semejantes a las que se presentan en casos similares
tratados con nefrectoma radical.
La linfadenectoma regional no mejora la supervivencia y nicamente
tiene, por tanto, validez en la estadificacin, por lo que no se realiza
sistemticamente. ste se establece sobre la base de los hallazgos qui

Figura 1 1 . R M de tumor renal con trombo en venas renal '1 cava

rrgicos y anatomopatolgicos.
Otras formas de tratamiento carecen de eficacia. Tanto la quimiotera

La arteriografa renal, exploracin obligada hace aos, ha quedado re

pia como la radioterapia ofrecen resultados pobres. En el caso de en

legada a los casos dudosos, riones nicos y otras situaciones en las

fermedad metastsica, las opciones son mltiples, pero ninguna satis

que se plantea tratamiento quirrgico conservador. El patrn arteriogr

factoria. Aunque se ha descrito la regresin de las lesiones metastsicas

fico caracterstico incluye neovascularizacin tumoral, lagos venosos,

tras la nefrectoma, esto ocurre nicamente en u n 1 %, Y generalmente

fstulas arteriovenosas y vasos capsulares.

de forma transitoria, por lo que no se justifica, salvo de forma paliativa


por otros motivos.

El estudio de extensin, si se sospechan metstasis, se completar rea


La inmunoterapia con i nterferones, interleucina, l infocitos killer acti

lizando radiografa de trax, analtica heptica completa y, en algunos


casos dudosos, gammagrafa sea.

vados, y ciertas combinaciones de quimioterapia con inmunoterapia,

El procedimiento diagnstico ante la presencia de masas renales se

ellas se obtienen tasas de respuesta superiores al 1 5%. Actualmente

puede observar en la Figura 1 2.

se utilizan de preferencia: antitirosincinasas en primera lnea, an

son alternativas para la enfermedad metastsica, pero en ninguna de

tiangiognicos en segunda lnea. Las situaciones que favorecen l a


respuesta al tratamiento i n m unoteraputico son: presencia d e me
MASA RENAL descubierta accidentalmente
. Examen fsico

. Anlisis de orina

I
t

tstasis pulmonares exclusivamente, buen estado general, y que se


haya realizado la nefrectoma a n tes del descubrimiento de las masas
pul monares .
En la actualidad se i nvestiga sobre autovacunas elaboradas con linfoci
tos peritumorales que parecen ofrecer resultados alentadores.

Quiste complejo

Quiste simple

o masa slida

Observar

4.2. Otros tumores


Tumor d e Wilms (vase Seccin de Pediatra ) .
Tumores renales metastsicos. Pueden encontrarse metstasis en el

rin de tumores de pulmn (la ms frecuente), mama, mela nomas


Masa slida
o quiste complicado

Angiomiolipoma

/
NEFRECTOMIA
RADICAL
O PARCIAL

No complicado
OBSERVACiN

e infiltracin por linfoma.


Tumores benignos:
Adenomas corticales: son los tumores ms frecuentes del adulto,

Complicado

Nefrectoma simple

Nefrectoma parcial

Embolizacin

Figura 12. Algoritmo diagnstico de las masas renales

aunque indistinguibles clnicamente del adenocarcinoma, por lo


que se tratan como tales. E l criterio clsico de tamao (3 cm)
para su diagnstico diferencial no es vlido en la actualidad.
Angiomiolipomas: se asocian a la esclerosis tuberosa en un 50%
(M I R 98-99F, 146). Compuestos de una proporcin variable de
grasa, vasos y fibras musculares. Cuando son grandes (mayores

17

Manual ero de Medicina y Ciruga, B.a edicin


de 4 cm), pueden ocasionar u n sndrome de Wnderlich por

guirlo del adenocarcinoma, pero en la mayora de los casos,

sangrado relroperitonea l . Cuando se asocian a esclerosis tubero

ni stos ni la citologa o la biopsia, ofrecen garantas sufi

sa, suelen ser mltiples y bilaterales, por lo que deben tratarse de

cientes de su benignidad, por 10 que tienden a ser tratados

forma conservadora.

mediante nefrectoma.

Oncocitoma: considerado benigno, aunque en algunos se han

Nefroma mesoblstico (hamarloma fetal): es el tumor benigno

detectado metstasis. Hay criterios radiolgicos para dislin-

ms frecuente en recin nacidos y lactantes.

Casos clnicos representativos

Un hombre de 45 aos tiene un carcinoma de clulas renales extendido. los niveles


de GOl, fosfatasa alcalina, lOH y a-2 globulina son elevados y el tiempo de protom
bina alargado. El hgado aparece difusamente agrandado, pero no existen defectos
focales de infiltracin intraheptica. la explicacin etiolgica ms probable para
estos hallazgos ser:

1 ) los efectos hcpdlolxicos de tumor.

2) Metsl,lSis hep.ltica.

18

Un paciente de 62 aos, con alteracin de la funcin renal y crisis de hematuria,


presenta una masa abdominal palpable en nanco derecho. Se le realiza una le, de
tectndose una masa de carcter slido de 8 cm de dimetro en rin derecho. En la
anamnesis destaca que el paciente es fumador de 35 cigarrillos al da. Cul es, entre
los siguientes, el diagnstico de presuncin ms probable?

1 ) Nefroblasloma.

3) Amiloidosis.
4) Trombosis tumorales que obstruyen la vena heptica.
5) Hepatitis vrica aguda.

Uposarcoma.
3) Angiomiolipoma.
4) Adenocarcinoma..
5) Carcinoma epidermoide.

MIR 05-06, 105; RC: 1

MIR 99-00, 1 77; RC: 4

2)

05.

HIPERPLASIA y CARCINOMA
PROSTTICO
Aspectos esenciales

OrientacIn

MIR
Tanto la hiperplasia
prosttica benigna como
el cncer de prstata, son
dos temas fundamentales.
Probablemente el cncer sea
ms importante, sobre todo
en lo referente al tratamiento.
Hay que aprenderse muy bien
el resumen de la Tabla 1 1;
aporta muchas preguntas
acertadas a cambio de poco
esfuerzo.

[jJ

La hiperplasia prosttica benigna (HPB) suele afectar a la zona periuretral de la glndula. El cncer aparece
en la zona perifrica.
La hiperplasia prosttica benigna no guarda relacin con el cncer.
Tanto la HPB como el cncer tienen relacin con las hormonas sexuales, y suelen aparecer en varones
ancianos.

El tratamiento mdico de la HPB consiste en a-bloqueantes (relajan la musculatura uretral y del cuello
vesical), inhbidores de la 5 a-reductasa (disminuye el tamao glandular) y (toterapa. Esta ltima no ha
demostrado utilidad con parmetros objetivos.
El tratamiento definitivo de la HPB es la ciruga, que puede consistir en reseccin transuretral o en ciruga
abierta, dependiendo del tamao prosttico.
El cncer de prstata es casi siempre un adenocarcinoma, con gran frecuencia multifocal.
El cncer de prstata cada vez se diagnostica con ms frecuencia en fase asintomtica. Cuando presenta
clnica, puede consistir en sntomas urinarios similares a la HPB.
El PSA elevado no es diagnstico de cncer de prstata. Puede corresponder a una HPB. El diagnstico defi
nitivo de cncer prosttico precisa una biopsia.
Las metstasis lumbares son tpicas del cncer de prstata, pudiendo producir compresin medular.
El tacto rectal revela una prstata ptrea e irregular en el cncer de prstata. Sin embargo, al principio puede
no ser palpable, ni visible en la ecografa (T1).
La principal complicacin quirrgica del cncer de prstata es la impotencia.
Ante un sndrome de compresin medular por cncer de prstata, nunca se deben emplear anlogos de la
LHRH nicamente. Siempre deben asociarse antiandrgenos.
En el cncer de prstata, la indicacin ms clara de prostatectoma radical es el estadio T2a.
El tratamiento fundamental del cncer de prstata diseminado es la hormonoterapia.

5 . 1 . H i perplasia prosttica ben i g na


La hiperplasia prosttica benigna (HPB) afecta en mayor o menor grado a la gran mayora de los varones a partir
de la quinta dcada de la vida, alcanzando el 80-95% de la poblacin masculina de 80 aos.
La prstata se divide clsicamente en cinco lbulos (anterior, medio, posterior y dos laterales); aunque stos ni

iIl

camente se encuentran como tales en la edad fetal. En el adulto se puede interpretar la anatoma de la prstata
dividida en dos partes: una zona perifrica, donde se origina principalmente el carcinoma, y una zona periure

Preguntas

MIR 08-09, 106


- MIR 06-07, 1 03, 233
- MIR 05-06, 106
- MIR 04-05, 1 06, 225
- MIR 03-04, 91
- MIR 02-03, 188
- MIR 01 -02, 1 04, l OS, 1 07
- MIR aQ-Q1 , 1 1 9
- MIR 98-99, 1 3 7
- M I R 98-99F, 147

tral o transicional, de la que procede la HPB (Figura 1 3) .


La HPB est compuesta de una proliferacin variable de elementos glandulares, musculares y del estroma, que en
su crecimiento comprimen la prstata perifrica, formando la l l amada cpsula quirrgica. Su etiopatogenia no est
clara; aunque el estmulo andrognico a travs de su forma activa, la dihidrotestosterona, es fundamental, su papel

exacto no ha sido determinado. Las teoras ms recientes abogan por un desequil ibrio hormonal de estrgenos/an
drgenos, o fX'r la existencia de factores de crecimiento prostticos con un papel permisivo del ambiente hormonal.
No existe evidencia de asociacin entre HPB y carcinoma prosttico.

19

Manual eTO de Medicina y Ciruga, B.a edicin


Fase clnica (Figura 1 5). La elongacin de las fibras musculares por

Estroma fibromuscular anterior

\?7

encima de un lmite condiciona prdida de capacidad contrctil. En


este momento aparece retraso del inicio de la miccin, disminucin
del calibre y de la fuerza del chorro micdonal y alargamiento del
vaciado (10 que en conjunto se denomina sndrome prosttico), El va
ciado suele ser incompleto, dando lugar a un residuo postmiccional.

--=,- Zona perifrica

zona central

Glndulas

Lbulo anterior

suburetrales

Conductos
eyaculadores

Lbulo
lateral

Lbulo posterior

Figura 13 . Anatomra de la prstata

Urografa lntraveno mostrando Impronta prosttica en vejiga (vejiga en montera")


con urteres en anzuelo

Diagnstico

Figura 1 S. Hiperplasia prosttica en fase clnica

Fase de descompensacin (Figura 1 6). Se produce un vencimiento

El crecimiento prosttico generalmente se produce hacia la uretra,


ocasionando obstruccin de sta y dificultando el vaciamiento vesical

del detrusor vesical, que es incapaz de vencer la presin uretral, au

(Figura 1 4). Esto no se manifiesta inmediatamente, sino que, general

cin urinaria. Ocasionalmente puede aparecer dilatacin ureteral

mentando la sintomatologa anterior y pudiendo presentarse reten

mente, el proceso pasa por una serie de etapas que incluyen una fase

bi lateral con deterioro de la funcin renal. Esto se debe a uropata

de compensacin, una fase clnica y una de descompensacin.

obstructiva infravesical con prdida del mecanismo antirreflujo.

"

19

e"

E
o
e

'2

u
>
.
,

c.
"
"I

Figura 14. Ecografa de hiperplasia prosttica con crecimiento


del 16bulo medio intravesical

Fase de compensacin. El crecimiento prosttico ocasiona un au

Situacin del mismo paciente un ao despus de la anterior: hidronefrosis grave.

De la vejiga (no se observa) se evacuaron 3500 ce de orina

Figura 16. Hiperplasia prosttica en fase de descompensaci6n

mento de la presin uretral durante el vaciado que es compensado

20

por una mayor actividad contrctil del detrusor que se hipertrofia,

Pueden producirse tambin otro tipo de sntomas denominados "irrita

encontrando presiones vesicales ms elevadas. En esta fase, la clni

tivos", que son debidos a la alteracin funcional vesical, y cuya reso

ca puede ser mnima o inexistente.

lucin es ms difcil tras la desaparicin de la obstruccin (MIR 01 -02,

u rolo 9ia

1 04). Entre estos sntomas se incluyen polaquiuria, tenesmo, nicturia y

doxazosina, terazosina, tamsulosina, etc.) que relajan la musculatura

urgencia mlccional. La HPB es la causa ms frecuente de obstruccin

del cuello vesical y uretra. Hasta ahora, estos frmacos se han estado
utilizando en forma de escalera teraputica, pero la aparicin del es

del tracto urinario inferior en el varn.

tudio COMBAT parece indicar que en pacientes con sintomatologa a


En la evaluacin del sndrome prosttico, el tacto rectal contina sien

partir de moderada, y con volmenes prostticos por encima de 30-40

do la exploracin fundamental, sobre todo para diferenciarlo del car

ce se debe realizar terapia combinada de inicio.

cinoma, ya que no es infrecuente que ambas entidades coexistan. La


clnica es lo ms importante para valorar la indicacin de tratamiento
de la HPB, ya que no existe correlacin entre el tamao prosttico y el
grado de obstruccin (MIR 08-09, 1 06). Cualquier zona sospechosa al

o RECUERDA

La finasterida tambin es til para la alopecia androgni ca, donde se


emplea en dosis mucho menores.

tacto debe ser biopsiada.


La medicin del flujo mximo miccional es tambin importante, conside

Como inconvenientes principales de los inhibidores de la 5 a-reductasa,

rndose normal cuando es mayor de 1 5 mi/s y claramente patolgico si

se encuentran: i mpotencia, reduccin del PSA en torno al 50% (difi

es menor de 1 0 mi/s. El estudio puede completarse con una ecografa que

cultando el diagnstico del carcinoma, si lo hubiese) y que tarda una

permita evaluar si existe afectacin del tracto urinario superior, residuo

media de cuatro meses en hacer efecto.

postmiccional, litiasis vesical u otra patologa asociada. El uso del PSA en


la HPB nicamente est indicado para descartar la presencia de carcino

De los a-bloqueantes, el inconveniente principal es la hipotensin.

ma en la prstata, ya que no sirve para diagnosticar HPB, aunque recien


temente ha demostrado ser el mejor predictor de la historia natural de la

En cuanto a las indicaciones de tratamiento quirrgico, globalmente,

enfermedad. Es decir, que mayores niveles de PSA en HPB diagnosticada

slo un 1 0% de los pacientes prostticos precisar ciruga. La inten

probablemente se correlacionarn con mayores volmenes prostticos y


con ms posibilidades de complicacin derivadas de la HPB.

sidad de las manifestaciones clnicas subjetivas y la mala respuesta al


tratamiento mdico pueden constituir la indicacin para la interven
cin. Entre las causas "objetivas" que suponen indicacin absoluta de
tratamiento quirrgico se encuentran (MIR 01 -02, 1 05):

Trata miento

Retencin urinaria reiterada.


Hidronefrosis retrgrada (lesin del parnquima renal por obstruc
cin infravesical).

Dentro de las posibilidades teraputicas, la ciruga contina siendo el


nico tratamiento definitivo para la HPB. tsta puede ser endoscpica

Infeccin urinaria de repeticin.

(RTUp: reseccin transuretral prosttica) o abierta (adenomectoma pros

Hematuria de repeticin.

Litiasis vesical.

ttica) (Figura 1 7), dependiendo del tamao del adenoma. En el 1 0% de


las piezas obtenidas se encontrarn focos de adenocarcinoma incidental.

5.2. Carci noma p rosttico


El adenocarcinoma prosttico es el tumor maligno ms frecuente del
aparato genitourinario masculino y el segundo en frecuencia general,
despus del pulmonar. Sin embargo, si se incluyesen los carcinomas
incidentales y los encontrados en autopsia, supera al pulmonar en pre
valencia (MIR 06-07, 1 03).
La hormonodependencia del cncer prosttico parece indicar el papel
de los andrgenos en su etiologa o patogenia. La relacin de factores
genticos, ambientales o infecciosos no ha quedado suficientemente
establecida.
El 95% de los carcinomas prostticos son adenocarcinomas originados
en la zona perifrica de la prstata. Los carcinomas ductales se originan
Figura 17. Adenomectomia prost ti ca

Se debe tener en cuenta que en la ciruga de la HPB no se extirpa la

en los conductos prostticos en lugar de los acinos, e histolgicamente


pueden corresponder a carcinomas transicionales, escamosos, endome
trioides o mixtos. Ms raros son los carcinosarcomas (menos del 1 %).

cpsula quirrgica, que est constituida por las glndulas prostticas

El adenocarcinoma prosttico, con frecuencia, es multifocal y presenta

perifricas comprimidas por el adenoma, y es el principal origen del

poblaciones en distinto grado de diferenciacin. En esta heterogenei

carcinoma prosttico, por lo que la intervencin quirrgica no protege

dad se basa la clasificacin de Gleason, que asigna una puntuacin de

del desarrollo de este proceso.

1 a 5, segn el patrn histolgico de cada una de las dos poblaciones

Los tratamientos no quirrgicos incluyen una variedad de fitoterapias,

ms representativas de la masa, sumando ambas puntuaciones para


obtener un resultado final de 2 a 1 0. Esta escala de Gleason se corres

poco efectivas si se valoran con parmetros objetivos, inhibidores de la

ponde con el pronstico de la enfermedad, independientemente del

5 a-reductasa (finasterida, dutasterida) (MIR 04-05, 225) que reducen el

estadio (MIR 06-07, 233). Para la estadificacin se emplea principal

tamao prosttico, antagonistas a-adrenrgicos (alfuzosina, prazosn,

mente la clasificacin TNM (Tabla 1 0 y Figura 1 8) (MIR 98-99F, 1 4 7).

21

Manual eTO de Medicina y Ci rug a, s.a edicin

Cl nica
. T1: tumor naparente cI(nicamente (no pal pable ni visible por tcnicas de
i magen)

T1 a: hallado incidentalmente. Afectacin menor del 5% del tej ido resecado


T l b: hallado incidentalmente. Afectacin mayor del 5% del tejido resecado
Tlc: Tumor identificado por p u ncin-biopsia por aumento del PSA
T2: tumor confinado a la prstata (in cl uye la inva sin de la cpsula prosttica
sin exteriorizacin del tumor hacia el tejido adiposo periprosttico)
T2a: menos del 50% de u n lbulo
- T2b: ms del 50% de un lbulo
- T2c: dos l bulos
T3: extensin del tumor por fuera de la cpsula
- T3a: extensin transcapsular (sea unilateral o bilateral)

El carcinoma prosttico es una enfermedad ms frecuente en ancia


nos, y la mayora de ellos se diagnostica por encima de los 60 aos.
Clnicamente puede producir sntomas obstructivos del tracto urina
rio inferior superponibles a los de la HPB. A stos puede aadi rse la
hematuria. El 25% de los pacienles que refieren retencin urinaria
aguda presentan u n carcinoma prosttico. Aproximadamente u n 25%
de los pacientes presentan metstasis en el momento del diagnstico;
stas pueden producir man ifestaciones como dolor seo, compresin
medular, mieloptisis o coagulopata. Afortunadamente, estos casos

- T3b: invasin de la(s) veslc u l a (s) seminal{es)

se encuentran en claro descenso gracias a la i ncorporacin del PSA

T4: tumor fijo o invade rganos adyacentes distintos a las vesculas


sem inales (cuello vesical, esfnter externo, recto, msculo elevador
o pared

(prostate-specific a n tigen -a n tgen o prosttico especfico), facilitando

el diagnstico de la enfermedad en estadios tempranos y comnmen


te asintomticos.

NX: no se pueden estudiar los ganglios regionales

NO: no metstasis ganglionares


N 1: metstasis a

Diagnstico

MO: no metstasis
M l : metstasis a distancia
Ml a: ganglios linfticos no regionales
Mlb: h ueso
M 1 c: otras local izacion es

Tacto rectal

Contina siendo el mtodo fundamental de cribado. Son accesibles al


tacto rectal todos los estadios excepto el T1, que por definicin es un

Tabla 1 0. Estadificacin del carcinoma de prstata

hallazgo. Caractersticamente, el carcinoma es duro, nodular e irre

T: tumor primario cUnlca TN

TX NO puede evaluar el tumor


TO No existen signos de tumor primario

na

Tla

na

Tlb

nb
Tlc

<5%

nb

>5%

T1 Tumor no evidente dlnic.amente, no palpable


ni visible mediante tcnicas de imagen:
na Extensin menor o igual a1 5% del tejido
resecado
. Tl b Extensin mayor del 5% del tejido
resecado
. TlcTumor identificado mediante puncin
bipsica (consecuenc.ia de un PSA elevado)

T3 Tumor que se extiende a travs de la

T2 Tumor limitado a la prstata o a la


capsula, sin sobrepasarla:
T2a Menos del 50% de un lbulo

capsula prosttica:
. TIa Extensin extracapsular (unilateral
o bilateral)
. TIb Tumor invade la veslcula seminal

T2b Ms del 50% de un lbulo


T2c Dos lbulos

N: ganglios linfticos regionales

T4 Tumor fijo o que invade estructuras adyacentes diferentes a las veslculas seminales

NX
NO

No se pueden evaluar los ganglios linfticos regionales


No hay metstasis ganglionares regionales

Nl

Metstasis en ganglios linfticos regionales

Figura 18. Estadificacin del adenoca rd noma de prstata

22

Ur0109a

guiar. En general, se aconseja un tacto rectal y un PSA anual a todos

tivo se ha i ntentado aumentar su especificidad para cncer con otros

los varones por encima de 50 aos aunque, de momento, la OMS no

parmetros (densidad de PSA, ndice PSNedad, velocidad de cambio

aconseja la realizacin de cribado poblacional sistemtico.

del PSA, PSA libre), aunque an no ha quedado establecida la ventaja


de stos sobre el PSA aislado.

Marcadores tumorales

Si el PSA es menor de 4 ng/ml, es poco probable que se encuentre un

Se dispone fundamentalmente de dos marcadores tumorales. La fosfa

cncer de prstata. Si es mayor de 1 0, las probabi lidades aumentan, lo


que aconsejara una biopsia de prstata ecodirigida. S i est entre 4 y

tasa cida prosttica (FAP) se emplea en clnica desde hace dcadas,


es un marcador especfico, pero su elevacin suele indicar extensin

1 0, se pueden utilizar los parmetros antes mencionados para valorar


la necesidad de biopsia (MIR 0 1 -02, 1 07).

extraprosttica, por lo que no resulta til en el diagnstico precoz. El


antgeno prosttico especfico (PSA) es realmente un marcador de te
jido prosttico cuyos niveles suelen encontrarse ms elevados en el

Pruebas de imagen

cncer, pero es inespecfico y tambin estn elevados a consecuencia


de patologa benigna (infecciones, sondajes, HPB, etc.). Por este mo

La ecograra transrectal (ETR) (Figura 1 9) es el mtodo de imagen ms


til para la esladificacin local, pudiendo ofrecer informacin impor
tante sobre la afectacin capsular, de vesculas seminales, cuello vesi
cal o recto. Aunque no existe un patrn caracterstico, suele aparecer
como ndulos hipoecognicos. La ecografa transrectal ofrece, ade
ms, la posibilidad de dirigir la biopsia hacia las zonas sospechosas.
La ecografa abdominal no tiene gran valor en la deteccin del car
cinoma prosttico. La TC y la RM tienen su principal papel en la es
tadificacin ganglionar y la valoracin de metstasis a distancia. Las
primeras metstasis deben buscarse a nivel de los ganglios l infticos de
las cadenas obturatrices e ilacas.

Gammagrafa sea

Se utiliza para la deteccin de metstasis seas, tiene mayor sensibi


lidad que la radiologa convencional (Figura 20), y debe realizarse en
todo paciente en quien se sospeche metstasis (Gleason > 8, PSA > 20).
Antes de plantearse el tratamiento curativo, en ciertos pacientes con al
tas probabil idades de encontrarse el cncer extendido, se debe efectuar
una gammagrafa previa para confirmar la no existencia de metstasis
seas o una TC para descartar metstasis ganglionares.

I) ETR corte transversal; {b} ETR corte longitudinal; (el Adenocarclnoma,


ndulo hlpoecojco en lbulo derecho

Figura 19. Ecografra transrectal (ETR) de adenocarcinoma prosttico

Figura 20. Radiografa de columna. Metstasis osteoblsticas

23

.a

Manual CTO de Medicina y Ciruga, 8," edicin

D RECUERDA

Agonistas LHRH. Aunque inicialmente ocasionan un aumento

las metstasis del cncer de prstata son osteoblsticas, es decir, for


man hueso (no en la gammagrafa, sino en la rad iografa) .

de los niveles de testosterona, posteriormente suprimen la se


crecin de lH y de andrgenos. la elevacin transitoria de los
andrgenos puede empeorar el cuadro clnico, principalmente
si existe compromiso medular por metstasis seas. Esta eleva
cin rflare-up) se debe suprimir mediante la administracin de

Biop sia prosttica

antiandrgenos, previamente a la introduccin de inhibidor de

Debe realizarse para la confirmacin del diagnstico. Puede efectuarse

Antiandrgenos (bicalutamida, flutamida, acetato de ciprotero.

la LHRH (MIR 00-0 1 , 1 1 9).


va transrectal o transperineal, guiada por el tacto rectal o bien guiada

na). Compiten con el receptor andrognico. Suelen utilizarse con

por la ecografa transrectal, lo que aade efectividad a la prueba. la

con agonistas de la lHRH. El acetato de ciproterona, adems de


actuar como antiandrgeno, tiene un efecto progestgeno, por lo

realizacin de la biopsia est indicada siempre que exista una anoma


la del tacto rectal, elevacin de los marcadores tumorales o alteracin
en las pruebas de imagen. la puncin-aspiracin con aguja fina (PAAF)

que acta a nivel central, disminuyendo los pulsos de lH.

es una alternativa con menores complicaciones, pero con el inconve

ttico. Se han realizado tratamientos con frmacos, que son una

niente de que no puede evaluar el grado histolgico (Gleason).

Quimioterapia. No es muy efectiva en el adenocarcinoma pros

mezcla de un estrgeno y una mostaza n itrogenada (fosfato de


estramustina), pero los estudios son contradictorios y las res

D RECUERDA

puestas pobres. Existen estudios prometedores con el empleo de

Son indicaciones de b iops ia prosttica el tacto rectal sospechoso, la


presencia de un ndulo ecogrfico y un PSA > 4 (variable la cifra segn
criterios).

docetaxel en pacientes con tumores hormonorresistentes.

Tratamiento por estadios (MIR 04-05, 1 06)


Estadio Tla. Tienen una mortalidad por la enfermedad del 2% a los

Trata miento

1 0 aos, por lo que no precisan tratamiento, salvo quizs los pacien


tes jvenes (menores de 60 aos) con una elevada esperanza de vida.
Estadio Tlb-Tlc. Alcanzan una mortalidad del 80% dejados a su

Opciones teraputicas

evolucin natural. Por ello est indicada la prostatectoma radical,


la radioterapia externa o braquiterapia, en sujetos con esperanza de

Prostatectoma radical. los pacientes candidatos deben ser individuos

vida superior a 1 0 aos (MIR 02-03, 1 88) .

con una esperanza de vida superior a 1 0 aos. Como complicaciones,

Estadio T2a. Es la indicacin ms clara de prostatectoma radical. la

se puede encontrar incontinencia (2-57%), estenosis anastomtica


(1 0%), impotencia (50%) o incluso la muerte 5%). En l neas gene

radioterapia o braquiterapia se reservara para pacientes de riesgo


quirrgico elevado o que no aceptan efectos secundarios atribuibles

rales, suele ir acompaada de linfadenectoma leo-obturatriz.

a la ciruga.

Radioterapia. Como tratamiento curativo los resultados en estadios

localizados se acercan a los de la ciruga. la diarrea crnica, la

Estadio T2b y T2c. Un 40% demuestra ser en realidad estadio 3, tras


el anlisis de la pieza qu irrgica de prostatectoma radical (infraes

proctitis, la cistitis rdica y las fstulas urinarias son complicaciones

tadificacin). la radioterapia externa o braquiterapia tambin puede

del tratamiento, as como la incontinencia y la impotencia a partir

ser til en pacientes de alto riesgo quirrgico.

de los dos aos de tratamiento.

Estadio T3a. la i ndicacin quirrgica es dudosa, as como la radio

Se ha empleado tambin radioterapia intersticial (braquiterapia) con

terapia local, por lo que solamente se propondra a sujetos jvenes,

implantacin de yodo-1 23 (1- 1 23), oro-1 98 (Au-1 98), paladio e iri

aun a costa de obtener malos resultados. Generalmente son tratados

dio. Su indicacin queda limitada a tumores pequeos de estadio T1

como el grupo siguiente.

o T2, y sus resultados son similares a los de la ciruga. En caso de


compresin medular o dolor por metstasis seas, la radioterapia so

Estadio T3b, T4, N+, M+. Varn aoso con mal estado general. El

bre la metstasis puede conseguir el control local de la enfermedad.

uso de radioterapia paliativa sobre la metstasis en caso de dolor

Hormonoterapia. El adenocarcinoma prosttico est compuesto por

(MIR 03-04, 9 1 ; MIR 98-99, 1 3 7).

tratamiento hormonal es la opcin indicada. Puede ser preciso el

una poblacin heterognea de clulas andrgeno-dependientes y


andrgeno-independientes. La supresin hormonal frena el creci
miento de las primeras, pero no afecta a las a ndrgeno-indepen

Recidiva tumoral posterior a tratamiento con intencin curativa

dientes. Se puede conseguir disminuir los niveles de andrgenos


circulares por distintos mtodos:
Castracin quirrgica. Es el mtodo aislado ms eficiente, con la

Tras la realizacin de prostatectoma radical, los pacientes son monito


rizados generalmente con peticiones de PSA

ventaja de que elimina la necesidad de medicacin permanente.


Por su rapidez en el efecto supresor hormonal, tambin est in

Cuando las cifras de PSA tras prostatectoma radical son superiores a

dicada en las compresiones medulares por metstasis.


Estrgenos (dietilestilbestrol). Inhibe la secrecin de LH. Actua

0,4 ng/ml, se considera recidiva bioqumica y debe hacer sospechar la


existencia de metstasis a distancia, o bien la existencia de recidiva a

lemente este mtodo se ha abandonado debido al alto riesgo

nivel local (MIR 05-06, 1 06)_

cardiovascular que conlleva.


tiandrgenos, al unirse a los receptores de la dihidrotestosterona.

Tras la realizacin de radioterapia como tratamiento de cncer de


prstata localizado, los descensos de PSA van siendo paulatinos (a di

Es preciso aadir estrgenos para evitar el fenmeno de escape,

ferencia de la prostatectoma radical) hasta conseguir un valor nadir,

que se produce tras varios meses de tratamiento. No son de uso

que es el valor mnimo alcanzado tras el tratamiento y que se con

habitual.

siderar referencia para el seguimiento posterior. Existen diferentes

Progestgenos. Inhiben la secrecin de lH y actan como an

24

UrO,09a
I

criterios para considerar el diagnstico de recidiva bioqumica tras

"PB

tratamiento con radioterapia: cuando se constata la existencia de tres


elevaciones sucesivas a partir del valor nadir, cuando se evidencian
n iveles nadir+2 (criterio de la ASCO), o con valores nadir+3 (criterio
de Philadelphial.

Locotlucln

Zona translcional

Perifrica

Olnlca

Fases:
Compensacin
CHnica
Descompensacln

25% slndrome prosttico


25% retencin aguda
25% metstasis

Tratamiento de urgencia

La compresin medular por el cncer prosttico no tratado puede ser la


forma de presentacin y constituye una urgencia importante. El objetivo

OIognOstlco

Tacto rectal
Ecografla transrectal (estadificacin local)
Gammag rafla osea (metstasis seas)

d_.1

PSA (muy sensible, poco especifico). Descarta cncer prosttico,


pero no diagnostica HPB

Tratamiento

Fitoterapia
Frmacos: finasterida,
n-bloqueantes
Clrugla: adenomectomla:
endoscplca o abierta

Fosfatasa cida (muy especifica, poco sensible)


Biopsia (confirmacin)

del tratamiento debe ser la supresin andrognica rpida o la descom


presin medular mediante laminectoma quirrgica o radioterapia. Se
pueden disminuir los niveles de andrgenos mediante castracin quirr
gica urgente, ketoconazol en altas dosis o dietilestilbestrol i ntravenoso.

ADENOCARCINOMA
PROSTATICO

A continuacin se expone una tabla que resume tanto las caractersti

Localizado: prostatectomla radical


ms Ilnfadenectomla bilateral.
radioterapia
Avanzado: castracin: quirrgica
(eleccin), farmacolgica

Tabla 1 1 . Tabla-resumen de las caracterfstlcas de la HPB


y del adenoca rcJnoma prosttico

cas de la hiperplasia prosttica benigna (HPB) como las del adenocar


cinoma prosttico (Tabla 1 1 l.

Casos clnicos representativos

Paciente de 66 aos, intervenido de prostatecloma radical, hace 3 aos por ade


nocarcinoma de prstata Cleason 8 (pT2b NOMO). Presenta, en el momento actual,
una cifra de PSA srico de 1 2 nglml. Seale cul de las siguientes afirmaciones le
parece correcta:

1)
2)
3)
4)
5)

La supervivencia media en el momento actual es menor de 1 ao.


La cifra de PSA est en rango normal ya que existen otras fuentes de produccin
del mismo.
El paciente puede tener una recidiva local o bien melstasis a distancia.
La utilizacin de bloqueo hormonal en este caso no es una opcin de tratamiento
posible.
En caso de tratarse de una recidiva local, estara indicado realizar ciruga de res.
cate para extirpar dicha masa.

Un paciente de 67 aos acude a Urgencias por presentar en los ltimos das de


bilidad progresiva de miembros inferiores, dificultad miccional e incontinencia
fecal. En la exploracin fsica destaca cierta hipotona anal, con una prstata muy
sugerente de malignidad al lacto, y debilidad de extremidades, conservando la sen
sibilidad tctil. Con el probable diagnstico de carcinoma de prstata melastsico,
cul de las siguientes opciones considera MENOS indicada para el tratamiento
de urgencia?

1 ) Estrgenos intravenosos.

2) Anlogos LHRH.

3) Ketoconazol (altas dosis) .

4) Radioterapia.
5) Orquiectoma bilateral.

MIR 05-06, 106; RC: 3

MIR 00-01 , 1 1 9; RC: 2

Hombre de 77 aos que refiere clnica de prostatismo de aos de evolucin, y que


presenta elevacin del PSA (antgeno prosttico especifico) (89 ng/ml) y dolor en
columna lumbar desde hace 2 meses. Al tado rectal, la prstata est aumentada de
tamao, de consistencia dura en ambos lbulos, superficie nodular y lmites mal defi
nidos. Tras realizarle una ecografa transrectal con biopsias prostticas ecodirigidas,
es diagnosticado de un adenocarcinoma de prstata pobremente diferenciado, que
afecta a ambos lbulos y que infiltra las vesculas seminales. La gammagrafa sea
confirma la presencia de metslasis en columna lumbar. Qu tratamiento de los
siguientes aconsejara en primer lugad

A la consulta acude un paciente de 54 aos con molestias a la miccin. Refiere


disminucin del c,",orro, dificultad para el inicio, goteo postmiccional, sensacin de
tenesmo y nicturia de tres veces. Presenta cultivos negativos y PSA de 2,1. En la
ecograffa abdominal se objetiva una glndula prosttica de 43 cc. Al tacto reclal
no se palpan ndulos sospechosos. En el IPSS obtiene una puntuacin que permite
clasificar su sintomatologa de moderada-grave. Su actitud deber ser:

1 ) Prostalectoma radical.
2) Quimioterapia intensiva.
3) Hormonoterapia.
4) Radioterapia pelviana externa.
5)

Braquiterapia prosttica.

1) Debido a su edad, el primer paso ser iniciar tratamiento con Htoterapia.


2) Debido a la gravedad de los sntomas se debe plantear ciruga de entrada.
3) Se debe iniciar tratamiento con a-bloqueantes.
4) La mejor opcin ser i nicia r tratamiento combi nado con a-bloqueantes+inhibidores
de la 5 a-reductasa.
5) Se debe iniciar tratamiento con inhibidores de la 5 a-recluctasa.
Re: 4

MIR 03-04, 9 1 ; RC: 3

25

06.
CARCINOMAS
DEL TRACTO URINARIO

Aspectos esenciales

Onentacln

MIR
Las preguntas sobre este
tema suelen ser senciHas

y repetitivas, aunque
ltimamente ha aparecido

como nuevo concepto el


carcinoma in silu. El estudio
del Desglose es especialmente
importante, pero se debe
tener en cuenta que la
tendencia parece orientada
a preguntarse cada vez ms.
Es fundamental la parte de
tratamiento.

GJ

El carcinoma vesical ms frecuente es el urotelial, siendo el tabaco el principal factor de riesgo.

(Il

El carcinoma escamoso se relaciona con la esquistosomiasis (5. haemafobium).

(I)

El adenocarcinoma vesical se relaciona con el antecedente de exLrofia vesical.

(i)

El carcinoma papilar superficial y el carcinoma in situ (OS) son muy recurrentes.

(I)

Clnica ms frecuente del carcinoma urotelial: hematuria, ms tpico con cogulos.

Cuando se trata de un carcinoma in situ: sntomas rritativos (polaquiuria, disuria, tenesmo, etc).

El mejor mtodo para la estadificacin local es la reseccin transuretral.

ill

Prueba ms sensible para el diagnstico de OS: citologa urinaria.

(2J

Conducta ante un ClS: tratar con bacilo Calmette-Guerin (BCG ) y revisiones (cistoscopia y citologas).

G:Q)

Actitud ante un tumor superficial: reseccin transuretral. Posteriormente, revisiones (CiSloscopia y citologas).

G:IJ

Actuacin ante un tumor infiltrante (afectacin capa muscular): cistectoma.

6 . 1 . Carci n o m a vesical
E l carcinoma vesical es l a segunda neoplasia urolgica e n frecuencia. Aparece ms frecuentemente en varones
(2-3 : 1 ) y ms en poblacin blanca que negra. Su edad de mxima incidencia se sita entre los 60 y 70 aos. De
el los, el 90% son carcinomas transicionales, el 8% escamosos y el resto adenocarcinomas. El epitelio uroteJial
recubre el tracto urinario desde las papilas caliciales hasta la uretra prosttica, ambas incl usive. En cualqu iera
de estos niveles pueden desarrollarse los tumores uroteliales, correspondiendo la mayora a la vejiga (ms del
90%) y, ms raramente, al tracto urinario superior (5%) o la uretra (1 %).
Entre los factores etiolgicos (Tabla 1 2), se implican las ami nas aromticas, presentes en las industrias textiles,
qumicas y del caucho. El humo del tabaco es el principal factor de riesgo (50-60% aparecen en fumadores),
aumentando el riesgo a mayor consumo. Tambin pueden jugar un papel importante los edulcorantes artificiales
(sacarina, ciclamato), la ciclofosfamida, los acetiladores lentos (mayor riesgo) y muchas otras posibles etiologas.
La infestacin por Schistosoma haematobium aumenta la incidencia de carcinoma escamoso vesical, as como
la presencia de infeccin crnica o catter vesical permanente.

eL

Preguntas

- MIR 07-08, 102


- MIR 06-07, 96
- MIR 05-06, 103
- MIR 04-05, 259
- MIR 03-04, 82
- MIR 02-03, 1 79
- MIR 00-01 , 1 1 8
- MIR oo-O I F, 1 43, 144
- MIR 98-99, 136
- M I R 98-99F, 1 4 5

26

Aminas aromtlcas {2-naftllaminal; tabaco, industria


textil, industria del caucho, colorantes
Fenacetinas crnicas
Sacarina, ciclamato
Ciclofosfamida (acrolelna)
Tabaco: ortofenoles, trlptfano

Translclonales (90%)
Mejor pronstico

Schfsrosoma haematobium
litiasis, infecciones, catteres

Escamosos (8%)

Cistitis glandular
Extrofla vesical

Adenocarclnomas (2%)
Tabla 12. Factores etiolgicos de los carcinomas del tracto urinario

UrOI09a

El adenocarcinoma primario vesical es un tumor raro, aunque es el que


se ha visto asociado a la extrofia vesical con mayor frecuencia.

o RECUERDA

No hay que confundir Schisrosoma haemarobium con SchislOsoma

mansoni, que prod uce

hipertensin portal.

Histologa e historia natural


Haciendo referencia al carcinoma de clulas transicionales, hay que di
ferenciar tres formas de la enfermedad con comportamiento, pronstico
y tratamiento completamente distinto (Tabla 1 3 y Figura 2 1 ). El 70% de
los tumores vesicales se presentan como tumores papilares de crecimien
to principalmente endocavitario y frente de invasin nico. Un 1 0% son
slidos, con invasin tentacular en profundidad y extensin linftica y vas
cular temprana. El 20% restante son formas mixtas. El primer grupo suele
corresponder a tumores superficiales de bajo grado histolgico, mientras
que los slidos, con mayor frecuencia, son tumores infiltrantes de grado
histolgico ms elevado. la principal caracterstica de los tumores papi
lares superficiales es la recurrencia, que ocurre en un 50-75%, segn el
grado y estadio. El 25% recurrirn y progresarn en grado y estadio, y ni
camente e1 1 5% acabar desarrollando un tumor infiltrante o metastsico.
la mayora de los tumores infi ltrantes se encuentran confinados a la
vejiga en el momento del d iagnstico, y slo un 20-25% presentan

1. Mucosa ----;. Ta,Tis

2.5ubmucosa

T1

4. Grasa -----+J T3
5. rganos vecinos

T4

3. Muscular ----+J 12
Figura 2 1 . Esquema de la estadificacin del tumor vesical

Se puede encontrar en el mbito vesical disti ntas lesiones benignas que


no se asocian con el desarrollo de cncer: los nidos de von Brunn, la
cistitis qustica y glandular origi nados en procesos inflamatorios o rrila

extensin ganglionar o metastsica. El 50% desarrollarn metstasis a

tivos crni cos, y que probablemente sean distintas manifestaciones de


un mismo proceso, aunque pueden plantear el diagnstico diferencial;

distancia, a pesar del tratamiento (MIR 04-05, 259).

(ocasionalmente se han descrito adenocarcinomas vesicales asociados

El tercer grupo que merece mencin aparle es el carcinoma in situ. A

a la cistitis glandular). Olras lesiones benignas seran el adenoma nefro


gnico, el plipo simple, el papiloma i nvertido o el papiloma velloso.

pesar de encontrarse l imitado al urotelio, por lo que es superficial, est


formado por clulas poco diferenciadas con displasia grave. Tiene una
alta tasa de recidiva y progresa hacia tumor i nfiltrante en el 50-75% de
los casos. Este mal pronstico le confiere un carcter completamente

Diag nstico

distinto del carcinoma in siw de otras regiones, en las que se considera


el estadio inicial de la enfermedad tumoral. El carcinoma in situ puede

la hematuria macroscpica o m icroscpica monosintomtica es el ha

estar asociado a focos de carcinoma superficial (26%) o nfiltrante (60%)

llazgo ms frecuente, presente en el 75% de los pacientes (MIR 06-07,

o bien encontrarse de forma aislada, siendo generalmente multifocal

96). la presencia de microhematuria asintomtica, descubierta durante

tanto en vejiga como en otras puntos del uratelio (MIR 07-08, 1 02).

estudios de cribado, slo se relaciona con enfermedad significativa en


menos del 2% de los casos. Pueden encontrarse sntomas irritativos
(escozor, polaquiuria, tenesmo) en el 25-30%, solos o acompaando a
la hematuria. la presencia de un sndrome cisttico no justificado por
i nfeccin o litiasis debe hacer sospechar la presencia de un carcinoma

Tis: carcinoma in situ (plano)


Ta: carcinoma papilar no infiltrante
T1: tumor que invade tejido conjuntivo subepitelial
T2: tumor que invade msculo
- T2a: tumor que invade la mitad interna
- T2b: tumor que invade la mitad externa
T3: tumor que invade tejido perivesical:
T3a: microscpicamente
- T3b: macroscpicamente (masa extravesical)
T4a: tumor que invade prstata, tero o vagina
T4b: tumor que invade pared plvica o pared abdominal

Nx: metstasis ganglionar regional desconocida


NO: ausencia de metstasis ganglionar regional
N1: metstasis a un solo ganglio entre 2 y 5 cm
N2: metstasis en un ganglio mayor de 5 cm o mltiples no mayores de 5 cm
N3: metstasis mayores de 5 cm

vesical, especialmente por su asociacin con el carcinoma in situ (MIR


00-01 F, 1 44). Con menor frecuencia, el paciente consulta por dolor en
flanco por obstruccin ureteral, plvico o por edema en miembros in
feriores (extensin l inftica). la exploracin fsica suele ser i rrelevante,
salvo en la enfermedad avanzada.
las citologas urinarias son una prueba sencilla y fiable que debe ser
rea l izada en todos los casos de hematuria asi ntomtica o sospecha de
tumor vesical. Su sensibilidad depende del grado de diferenciacin
del tumor vesical, alcanzando el 75-1 00% en tumores de alto grado
y carcinoma in situ, siendo en este ltimo un mtodo diagnstico ms
rentable que la ecografa, la Te, la urografa o la biopsia mltiple. Son
especialmente tiles en el seguimiento de pacientes sometidos a resec
cin transuretral en combinacin con la cistoscopia.
Entre las pruebas radiolgicas destacan la ecografa (con una sensibi l i

MO: no metstasis
M l : metstasis a distancia

Tabla 13. Estadificacin del carcinoma vesical

dad del 80%, pera poco til para el d iagnstico de las neoplasias del
tracto urinario superior (Figura 22]) y la urografa intravenosa (capaz de
detectar la presencia de tumor en el 60% de los casos).

27

Manual eTO de Medicina y Ciruga, B.a edicin


un 40%, siendo tambin tratamiento de eleccin en el carcinoma
in situ, con una eficacia del 70% (MIR 05-06, 1 03; M I R 02-03, 1 79;

MIR 00-0 1 , 1 1 8). Sin embargo, no se utiliza en los tumores vesicales


como primera lnea, reservndose para los tumores de riesgo o con
mltiples recidivas, ya que tambin se acompaa de mayor nmero
de efectos secundarios y de complicaciones, entre los que se en
cuentran cistitis febril, sndrome pseudogripal, y las ms graves de
sepsis, prostatitis granulomatosa, neumonitis e incluso muerte. Estos
cuadros precisan tratamiento tuberculosttico completo al menos de
tres a seis meses.
En la enfermedad i nvasora

infiltrante, tras la reseccin transure

tral para evaluar la afectacin parietal, el tratamiento de eleccin


es la cistectoma radical. La quim ioterapia sistmica, adyuvante o
neoadyuvante, no parece mejorar los resultados de la ciruga aislada.
Igualmente se puede decir de la radioterapia, que nicamente ha de
mostrado i ncrementar el tiempo libre de recidiva local, s i n aumentar
la supervivencia.
La quimioterapia combinada con M-VAC (metotrexato, vinblastina,
adriamicina, cisplatino) se reserva para pacientes en los que existe
En la urografa i ntravenosa, adems de la presencia de defectos de

afectacin ganglionar o metastsica, con respuestas completas en el

replecin, se puede sugerir el diagnstico de tumor vesical, rigidez

1 5-30% y parciales en el 30-40%. Tras la cistectoma, los urteres son

y falta de distensibilidad vesical, la obstruccin de un urter o el

derivados generalmente a segmentos intestinales o a piel, pudiendo

desplazamiento de la vejiga, entre otros. En caso de dudas sobre el

realizarse estomas no continentes o reservaras continentes directa

tracto urinario superior, se recurrir a la pielografa retrgrada en el


momento de realizar la cistoscopia. sta es fundamental para la eva

mente a l remanente uretral.

luacin del tumor vesical; puede real izarse bajo anestesia local cuan
do existan dudas con las pruebas realizadas previamente, pero si el
diagnstico de presuncin es firme, y dado que en todo tumor debe

o RECUERDA

la adriamicina (doxorrubicina) es un quimioterpico que puede produ


cir cardiotoxicidad.

realizarse reseccin transuretral para evaluar el grado de infi ltracin,


se puede esperar a tener al paciente en quirfano bajo anestesia gene
ral o raqudea para practicarla (MIR 00-01 F, 1 43). En un 1 0% de los
casos de cistoscopia con citologas positivas no se encuentra tumor
en la vejiga, lo cual puede ser debido a la presencia de carcinoma in
situ, tumor en vas urinarias altas, a un carcinoma ductal de prstata

o a un falso positivo de la prueba (generalmente por inflamacin de

6.2. Tu mores del tracto u r i n a rio


s u perior

la pared vesical o por tratamiento concomitante con radioterapia o


quimioterapia endovesical).
Entre e l 2-5% d e los tumores uroteliales se encuentran localizados
La Te se utiliza para la estadificacin de la enfermedad infi ltrante, ya

entre los cl i ces y los orificios ureterales. En su etiologa estn

que aporta informacin limitada sobre la infiltracin tumoral. La RM


consigue mejores imgenes de la cpula vesical por sus cortes sagita

i m p l i cados los mismos factores que para el carcinoma vesical, a


los que habra que aadir la nefropata por abuso de analgsicos

les, pero no aporta mayor informacin que la TC

(fenacetinas) y la nefropata de los Balcanes. En el 70-80% de los

Tratamiento

pacientes aparece hematuria macroscpica, siendo el dolor c


lico por obstruccin ureteral la segunda queja en frecuencia de
aparicin.

Todo el proceso diagnstico va encaminado a establecer si el tumor


vesical es superficial o infiltrante, ya que el tratamiento vara radical

o RECUERDA

la hematuria tambin es la manifestacin ms frecuente del hiperne


froma.

mente en funcin de este hecho.


Los tumores superficiales son manejados mediante reseccin transu
retral . Dada la alta frecuencia de recidivas, la mayora se tratan poste

Diag nstico

riormente con i nstilaciones endovesicales (quimioterapia o inmunote


rapia local) que disminuyan la aparicin de nuevos tumores. Entre los
quimioterpicos empleados se encuentran la mitomicina, la tiotepa, la

La sistemtica d iagnstica es bsicamente la misma que para el

adriamicina o la epirrubicina. Con todos ellos se consigue reducir las

tumor vesical. Es decir, ecografa (detectar hidronefrosis del lado

recidivas alrededor de un 20%.

del tumor), UIV (se ver un defecto de replecin no compatible

La inmunoterapia endovesical con BCG (bacilo Calmette-Guerin) es,

y Te para la estadificacin (Figura 23) ( M I R 98-99, 1 36; M I R 98-

sin duda, la ms eficaz, disminuyendo el porcentaje de recidivas en

99F, 1 45 ) .

con u n clculo o una anulacin funcional de ese sistema excretor),

28

UrOIOga

ureteropieloscopia, elemento que, adems de diagnstico, puede


tener un fin teraputico en tumores de pequeo tamao y aspecto
superficial (Figura 241.

Fumador + Hematuria

Sospecha d. tumor uroteUal

Citologias + Pruebas de imagen

<: ECO 'VP


UIV

No concluyente

Concluyente

para tumor vesical

para tumor vesical

-;:

t
:....'
t

:::;--l

Cistoscopia ES - =
=

. RTUV
'-=':=
:: =

6t

Biopsia vesical ES ----Tis


--.

6t

Sospechar tumor

Ureterorrenoscopia

Pielografa retrgada

Citologas selectivas

Cepillado ureteral

Revisiones
Cistoscopia
+ Citologa

ReCi iva TS

TA,Tl

BCG

Tracto urinario superior

2:12

Cistectomfa Quimio BCG (G3)

ClOmia

diva

Ftgura 24. Algoritmo diagostico-teraputko en tumor de vejiga (MIR 03-04,82)

Tratam iento
El tratamiento estndar es la nefroureterectoma radical con extirpacin
de un rodete perimetico vesical; tal extensin es necesaria por la ten
dencia a la recidiva de estos tumores. Es planteable, sin embargo, el
Figura 23. Ecografia.Tumor vesical con zonas calcificadas

empleo de tratamientos conservadores en caso de tumores papilares,


no infiltrantes, nicos y de pequeo tamao.

En estos tumores, la citologa urinaria aumenta su eficacia si se ob

Es necesario un seguimiento posterior de la vejiga y el rin contra la

tiene de forma selectiva, cateterizando el urter del lado afectado.

teral, ya que un 20-30% desarrollarn un tumor vesical metacrnico y

Otros elementos de diagnstico son las biopsias por cepillado y l a

un 2% en el sistema colector contralateral .

Casos clnicos representativos

Varn de 5 7 aos de edad, fumador, que consulta por hematuria terminal, polaquiu
ria, urgencia y dolor miccional. Presenta citologas urinarias positivas de carcinoma
urotelial, y el estudio anatomopatolgico tras la reseccin transurelral es de carci
noma in silu difuso, con intensa inflamacin crnica. El tratamiento estndar ser:

Mujer de 63 aos que es diagnosticada de carcinoma de clulas escamosas del Ir


gono vesical, con invasin de la capa muscular. Cul seria su actitud leraputica
en este caso?

1 ) Instalacin del bacilo de Calmette-Guerin.


2) Cistectoma radical.

2)

3)

Instilaciones con mitomicina.

4) Quimioterapia con cisplalino.

5) Antiinllamatorios no esteroideos ms quinolonas durante seis meses.

MIR 05-06, 1 03; RC: 1

1)

Radioterapia externa con 7.000 rads.

Quimioterapia adyuvante, seguida de cistectoma radical.


3) Reseccin transuretral, seguida de inmunolerapia intravesical (BCG).
4) Cistectoma radical con extirpacin de cara anterior de vagina.
5) Radioterapia externa, seguida de quimioterapia con cisplatino.
MIR 03-04, 82; RC: 4

29

Manual CTO de Medicina y Ciru g a, 8.... edicin

Casos clnicos representativos

Un paciente de 64 aos acude a consulta por sndrome miccional de dos meses de


evolucin. Refiere que sU MAP le ha dado tratamiento antibitico y le ha realizado
cultivos que han sido negativos. Se le realiza una ecografa y una flujometra que son
normales y una urografa intravenosa que no evidencia alteraciones. En la cistosco
pia no se observan lesiones intravesicales. las citologas urinarias son sugestivas de
malignidad. Seale la incorrecta
1)

El tratamiento de eleccin ser la inmunomodulacin vesical.

2) Es imprescindible la realizacin de biopsias para confirmacin del diagnstico.


3) Si recidiva tras las instilaciones, se deben repetir los ciclos dos veces ms.

4) la cistectoma es el tratamiento final en muchos de estos pacientes.


5) A priori no se encontrar infiltracin de la capa muscular.

RC: 3

30

Una paciente de 58 aos acude por polaquiuria de dos aos de evolucin, nicturia
y dolor hipogstrico. Niega haber visto sangre en la orina y es fumadora. Seale la
respuesta correcta:
1)

Se deben realizar biopsias a esta paciente.

2)

Una dstoscopia permitir confirmar el diagnstico.

5)

En todos los casos se detectarn > 1 0 hemates/campo en el sedimento de orina.

3) los tratamientos empleados hasta el momento han demostrado una gran eficacia.
4) Muy probablemente los cultivos sern positivos.

Re: 1

07.
TUMORES TESTICULARES

Aspectos esenciales

Ont>ntil(In

MIR
Estc tema es sencillo
y rentable siempre

que se seleccione
lo realmente importante:
la clnica y el diagnstico.
Es fundamental la revisin
profunda de los Desgloses.
En cuanto al tratamiento, dado
que no existe un protocolo
universalmente aceptado,
es mejor conocer ideas
generales.

El tumor testicular es la neoplasia slida ms frecuente en el varn joven.

la tasa de curacin es mayor del 90%.

[I)

El tumor testicular ms frecuente es el seminoma. No obstante, sto es muy variable segn la edad del pa
ciente.
Una masa testicular por encima de los 50 aos debe hacer pens.lr en un linfoma.

El tumor tpico de las disgenesias gonadales es el gonadoblasloma.


la clnica ms frecuente es una masa escrotal indolora.

El seminoma nunca produce afetoprotena .


Como tratamiento, la orquiectoma por va inguinal se realiza en todos los casos.
El seminoma es radiosensible. Por ello puede utilizarse radioterapia como tratamiento en los primeros esta
dios. Si se trala de un estadio avanzado, se empleara quimioterapia.
los lumores no seminomalosos se tratan con orquiecloma + quimioterapia. No obstante, si es un tumor
limitado al testculo, puede plantearse la vigilancia tras la orquiectoma.

7 . 1 . Eti ologa y epidemiolog a


Los tumores testiculares suponen el ' 2% de las neoplasias en varones, y son las neoplasias slidas ms frecuen
tes entre los 20 y 35 aos. Presentan mayor incidencia a mayor desarrollo de la sociedad, y tambin en la raza
blanca ms que en las dems. La tasa de curacin es superior al 90%.
El 95% de ellos proceden de clulas germinales y, aunque globalmente el semi noma es el ms frecuente, la in
cidencia vara segn el grupo de edad considerado. El 5% restante se reparte entre tumores del estroma gonadal
(1 2%), ! infamas (1 %), gonadoblastomas (clulas germinales y del estroma) metstasis y otros.
Los factores de riesgo para el desarrollo de tumor testicular son: teste h i poatrfico, sndrome de Klinefelter,
h i storia fam i l iar de neoplasias testiculares, tumor previo en el otro teste y criptorquidia. Los testculos no
descendidos tienen mayor riesgo de desarrollar tumores, y esta probabilidad aumenta si la situacin del
teste es intraabdominal. Asimismo, el testculo contra lateral, aunque de localizacin escrotal, tiene una
incidencia mayor; de hecho, el 20% de los tumores en pacientes con criptorquidia se desarrollan en el
testculo no criptorqudico. Por estas razones, los testculos no descendidos deben descenderse, preferi
blemente entre el primer y segundo ao de edad para facilitar su seguimiento y, con ello, la deteccin de
1

Preguntas

- MIR 09- 1 0. 102


- MIR 06-07, 94
- MIR 05-06. 107
- MIR 04-05, 107
- MIR 03-04, 145
- MIR 02-03, 186
- MIR 01 -02, 108
- MIR 00-01 F, 142
- MIR 99-00, 186
- MIR 99-00F, 145
- MIR 98-99F, 2 1 8

un eventual tumor.
Una vez alcanzada la pubertad, un teste criptorqudico no descendido probablemente deba extirparse, dado que
estos testculos pierden la capacidad de espermatognesis y conservan su potencial malignizante. No obstante,
hay autores que prefieren no extirparlos, siempre y cuando se puedan descender a la bolsa escrotal, ya que an
pueden mantener su funcin endocrina (secretora de testosterona) (MIR 9900, 1 86).
Otros factores que se pueden encontrar relacionados son las hernias inguinales infantiles (no demostrado) y
la orquitis urliana secundaria al paramixovirus causante de la parotiditis (siempre que haya producido atrofia)
y, entre los factores txicos, la exposicin a radiaciones, fuentes de calor, productos para teido del cuero y

31

Manual eTO de Medicina y Ciruga, 8.3 edicin


estrgenos ntra tero durante el primer trimestre del embarazo (MIR

saco vitelina en el 38% de los tumores testiculares del adulto.

98-99F, 2 1 8). Aunque entre un 8 y 25% de los pacientes presentan

Coriocarcinorna. En el estudio histolgico tiene que contener ele

historia de traumatismo testicular, todos los autores parecen estar de

mentos de sincitiotrofoblasto y citotrofoblasto para ser considerado

acuerdo en que ste supone ms bien el motivo por el que se descubre

como tal. En el momento del diagnstico, generalmente existen me


tstasis a distancia (suelen ser va hematgena) y un tumor primario

una masa escrotal, y no su origen.

testicular pequeo. Raramente es puro (MIR 06-07, 94).


Teratoma. Por definicin, se encuentra formado al menos por dos capas

7.2. Anatoma patolgica

distintas de clulas germinales (endodermo, mesodermo o ectodermo).


Teratocarcinorna. Tumor mixto con reas de teratoma y de carcino

ma embrionario. Un 64% tiene tambin reas de semi noma.


Conadoblastoma. Contiene grandes clulas similares al seminoma

Los tumores testiculares se suelen originar como una masa testicular

y otras menores, como clulas de Sertoli inmaduras o de la granulo

intraparenquimatosa. A partir de ah, pueden quedarse localizados o

sa. Se asocia preferentemente a las gnadas disgenticas y estados

metastatizar. La localizacin ms frecuente de metstasis son los gan

intersexuales.

glios retroperitoneales (casi siempre es el primer escaln en la disemi

Tumores de clulas de Sertoli. Forman reas tubulares similares a

nacin) posteriormente puede aparecer la afeccin mediastnica, de

los tbulos del testculo prepuberal normal. De comportamiento be

vsceras abdominales y de pulmn.

nigno, es excepcional que metastaticen.

Hay que tener en cuenta que el testculo izquierdo drena a los ganglios
paraarticos y prearticos a nivel de L2. El derecho drena a ganglios
interaortocavos, precavas y prearticos, y tambin a nivel del hilio re
nal. La diseminacin hematgena es menos frecuente, salvo en el co
riocarcinoma, va vasos espermticos, siendo los puntos ms habituales

RECUERDA

El lumor de clulas de Serlol i se ha asociado al sndrome de Peutz


Jeghers (vase Apa rtado de Sndromes de poliposis, en la Seccin de

Digestivo y ciruga general).

de metstasis: pulmn, hgado, hueso y SNC, por orden de frecuencia.


Tumores de clulas de Leydig. Pueden verse cristaloides en su cito

Los tumores testiculares se dividen en dos grupos: tumores que no deri

plasma. Generalmente son benignos. Pueden ser hormonal mente acti

van de las clulas germinales (5%) y tumores que derivan de las clulas

vos, produciendo pubertad precoz o feminizacin (MIR 99-00F, 145).

germinales (95%); a su vez, estos ltimos se dividen en seminomatosos

Linforna. Tanto como localizacin secundaria o como origen pri

y no seminomatosos (Tabla 1 4).

mario (menos frecuente), una masa testicular en un varn mayor de


50 aos sugiere en primer lugar el diagnstico de linfoma. Tras la

TUMORES DE CHULAS GERMINALES

TUMoRES DEL ESTRoMA

Tumonos de un tipo

$eminoma
- Tpico
- Anaplsico
- Espermatoctico
Carcinoma embrionario
PoHembrioma
Tumor del co vitelina (seno endodrmico)
Coriocarcinoma
Teratoma:
- Maduro
- I nmaduro
- Con transformacin maligna

orquiectoma o la biopsia testicular para el diagnstico de certeza,


el tratamiento no vara respecto a los l infomas habituales.

7.3. C l n i ca

Tumores clulas Leydig


Tumores clulas SertoH
Tumores estructuras
gonadales primitivas

Tumores de m6s de un tipo hIstcJI6gIco

Teratocarcinoma
Otros
Tumores germinales + estroma'"

Gonadoblastoma
Tabla 14. Tumores de testkulo. Clasificacin histolgica

La manifestacin ms frecuente es como masa escrotal indolora. Con


mucha menor frecuencia, el motivo de consulta est originado por la
presencia de metstasis ganglionares (masas supraclaviculares o abdo
minales (MIR 09-10, 1 02), o efectos endocrinos (ginecomastia, puber
tad precoz).
EI 1 0 % de los tumores se presentan como escroto agudo en la urgencia.

RECUERDA

Exislen algunas enfermedades, como

ta mao testicular sin existir un tumor.

la sarcoidosis, que aumentan el

Serninorna. Puede aumentar el tamao testicular hasta 1 0 veces sin

distorsionar su morfologa. Ocasionalmente se presenta extrates


tcular en mediastino (la ms frecuente), retroperitoneo, o regin
pineal de forma primaria. Se han descrito tres tipos histolgicos:

7.4. Diag nstico

seminoma tpico, anaplsico y espermatoctico (ms frecuente en


individuos mayores de 50 aos). El seminoma puro no es productor
de marcadores tumorales, pero hasta en un 1 5% de los casos pue

La exploracin fsica de una masa indolora, sin signos inflamatorios y

den aparecer clulas del sincitiotrofoblasto, produciendo elevacio

de largo periodo de evolucin, debe hacer sospechar un tumor testicu

nes de la p-HCG.

lar (Tabla 1 5).

Carcinoma embrionario. Tiende a metastatizar de forma temprana.

Forma masas ms irregulares y heterogneas que otros tumores.

32

Un elemento i mportante, tanto para su diagnstico como para el segui

Tumor del seno endodrmico. Este tumor fue descrito inicialmente

miento, son los marcadores tumorales: a-fetoprotena (AFP) y fraccin

como una forma rara de tumor infantil. Se encuentran elementos de

p de la gonadotropina corinica humana (P-HCG) (Tabla 1 6).

UrOIOga

presencia de enfermedad residual, ya que la elevacin persistente de


pTls
pTl
pT2
pTl
pT4
N1

Ganglios menores de 2 cm
Ganglios entre 2-5 cm
Ganglios mayores de 5 cm

N2
N3
M1.
M1b

Sx
SO
51

52
53

I ntratubular
Testculo y epididimo sin invasin vascular/linftica
Testculo y epiddimo con invasin vascular/linftica o tnica
vaginal
Afectacin de cordn espermtico
Escroto

Metstasis en ganglios no regionales o pulmn


Metstasis viscerales no pulmonares
Marcadores tumorales sricos no disponibles
Niveles de marcadores normales
lDH < 1.5 x n ; y p-HCG < 5.000; Y AFP < 1 .000
lDH entre 1,5 x n y 10 x n o p-HCG entre 5.000 y 50.000
o AFP entre 1 .000 Y 10.000
LDH > 10 x n o j3-HCG >50.000 o AFP > 1 0.000

uno de estos marcadores despus del tratamiento supone la existencia


de tumor no eliminado.
La ecografa testicular es un mtodo sencillo y fiable para la diferen
ciacin entre masas slidas y qusticas, y su localizacin exacta i ntra
testicular o dependiente de los anejos. Cuando, a pesar de los marca
dores, los datos ecogrficos son sugerentes de tumor, est indicada la
exploracin quirrgica a travs de una incisin inguinal, para evitar la
posibi lidad terica de implantes tumorales en la piel escrotal y tener
mejor control de pedculo vasculolinftico a nivel del cordn esperm
tico (MIR 01 -02, 1 08).
Si la exploracin confirma la presencia de una masa, el testculo debe
ser extirpado (orquiectomfa radical).
La evaluacin de la extensin tumoral se completar mediante TC to
racoabdominal. As se constatar si la enfermedad est limitada al tes
tculo (estadio 1), o existe afectacin de ganglios infradiafragmticos
(estadio 11), o bien si hay incluso extensin supradiafragmtica o a r
ganos slidos (estadio 1 1 1). El sistema de estadificacin utiliza mltiples
variaciones, pero quiz la clasificacin ms aceptada sea la expuesta
anteriormente en la Tabla 1 5 .

7 . 5 . Diagnstico d ife rencial


Tabla 15. Estadificaci6n clnica y patolgica

Aunque frecuentemente l a existencia d e u n tumor testicular n o plan


Tumor con una palabra aumenta ji-HCG

Coriocarcinoma
- Seminoma (en la mujer, el equivalente es el disgerminoma)
Tumor con dos palabras aumenta a.-fetoproteina

- Carcinoma embrionario
- Tumor del seno endodrmico

tea dudas diagnsticas, existen una serie de patologas testiculares


que, junto con los tumores, pueden presentarse en algn momen
to dentro del cuadro genrico que se denomina "escroto agudo",
caracterizado por el aumento doloroso de volumen del contenido
escrotal, acompaado o no de signos inflamatorios (Tabla 1 7 y Fi
gura 25).

Tabla 16. Marcadores tumorales en tumores germinales


(vlidos para ovario y testiculo)

Orquiepididimitis: suelen presentarse con dolor i ntenso, enro

La AFP es sintetizada por clulas del saco vitelino y, por tanto, est

disminuye el dolor (signo de Prehn positivo). En su etiologa se


deben considerar grmenes de transmisin sexual en pacientes

jecimiento cutneo, fiebre, y a veces si ntomatologa miccional.


En la exploracin, la elevacin del testculo (signo de Prehn)

presente en tumores de saco vitelino o seno endodermal, y en los


carcinomas embrionarios. El seminoma nunca produce AFP (MIR

adultos menores de 35 aos, y uropalgenos (E. coli) si superan

04-05, 1 07; M I R 02-03, 1 86). En cualquier caso, se debe tener en

esla edad.

cuenta que la AFP es u n marcador i nespecfico, y se podra en


contrar elevado en enfermedades hepticas benignas o malignas,

Torsin del cordn espermtico: suele aparecer en la infancia o

algunas neoplasias pancreticas y de la va biliar o en la ataxia

inflamatorios crecientes a medida que progresa el cuadro. El teste

telangiectasia.

se encuentra horizontal izado, y ocasionalmente puede palparse la

a RlECUpERDAb

.
.
'
'
.
a AF tam len se e Ieva en l IqUl-do amOlotlCO
en a IteraClones dei CIerre
del tubo neural. Ver Ginecologa.

la adolescencia, con dolor de aparicin brusca y signos cutneos

espiral del cordn torsionado. En este caso, la elevacin del testcu


lo i ncrementa la sensacin dolorosa.
Hidrocele y espermatocele: son dos cuadros que raramente se

presentan de forma brusca y con dolor agudo, fcil mente diferen

ciables por la exploracin y su transiluminacin positiva y, ante la


duda, mediante ecografa.
La fraccin p de la HCG es producida por las clulas del sincitiotrofo
blasto presentes en el coriocarcinoma, y tambin de forma ocasional y
de forma aislada, en algunos semi nomas (MIR 00-01 F, 1 42).
En conjunto, el 70% de los tumores testiculares producen algn marca
dor, luego existe hasta un 30% de tumores con marcadores negativos

Orquitis

Polaridad conservada
Prehn MEJORA el dolor

Torsin del cordn espermtico

Testculo horizontalizado
Prehn EMPEORA el dolor

Tumor testicular

Masa palpable indolora

al diagnstico.
La vida media de la a-fetoprotena es de siete das, frente a tres das
de la -HCG. Este dato es importante a la hora de valorar la posible

Tabla 17. Diagnstico diferencial de los tumores testiculares

33

Manual eTO de Medicina y Ciruga, 8.a edicin

D RECUERDA

El cisplatino produce vmitos con mucha frecuencia. Otro efecto secun

dario es su nefrotoxicidad.

Estadio lIa-lIb. En este caso, el tumor ya est extendido a gangl ios

retroperitoneales, y requiere, por tanto, tratamiento agresivo. Se


dispone de dos opciones: radioterapia sobre las cadenas afectas
(teniendo en cuenta el teste afectado, se irradiar a unas cadenas
ganglionares u otras), o quimioterapia BEP (cisplatino, etopsido y
bleomicina). Ambos obtienen resultados muy similares
Estadio 11e-1II (estadios avanzados). El tumor tiene metstasis gan

glionares retroperitoneales superiores a 5 cm, O afectacin supra


diafragmtica o de vsceras slidas. La radioterapia deja de ser una
opcin teraputica. La quimioterapia (BEP) es la nica posibilidad,
presentando una tasa de curacin de alrededor del 80%.

Tu mores no seminomatosos
Estadio 1. Existen tres posibilidades teraputicas tras la orquiectoma:

Observacin y seguimiento peridico. Tasas de recadas de un 20%.


linfadenectoma retroperitoneal de estadificacin, pues as se
tiene certeza del estadio y se reduce a un 1 0% las recidivas (en
Europa no est extendida esta prctica).
Quimioterapia profilctica (cisplatino), y as se reducen al 5%
Figura 25. Pieza

las recidivas.

de tumor testicula r

En aquellos casos en que exista invasin vascular en la pieza de


orquiectoma, la tasa de recidivas asciende al 50%. Parece lgico,
en estos casos, inclinarse de entrada por una de las dos ltimas op

7.6. Trata m i e nto

ciones. La tasa de curacin alcanza el 98%.


Estadio lIa-lIb. Histricamente se realizaba linfadenectoma retro

peritoneal completa como nico tratamiento, pero ante tasas de


Hasta la fecha no existe un protocolo nico de tratamiento, pudiendo

recidiva no desdeables, actual mente se indica quimioterapia de

variar incluso de un centro a otro. Siempre se realizar orquiectoma


radical va inguinal. Posteriormente, el patlogo informar de la estir

inicio (BEP). La tasa de supervivencia supera el 95%.


Estadio 11e-1II. Antes de la aparicin de la actual quimioterapia, la

pe histopatolgica. A continuacin, el estudio de extensin mediante

supervivencia era del 51 0%. Actualmente, el esquema de trata

Te toracoabdominoplvica y nuevos marcadores postorqu iectoma. En

miento mayoritariamente aceptado es quimioterapia primaria.

funcin de la histologa y del estadio, se asignar un tratamiento a cada


El tratamiento de la neoplasia testicular se resume en la Figura 26.

enfermo.

D RECUERDA

La orquiectoma en el cncer de testculo


de prstata , va escrotal.

es

TUMOR TESTICULAR

v a inguinal. En el cncer

Orquiectoma radical

Seminoma

NO SEMINOMA

SEMINOMA

Se caracteriza por su gran radiosensibilidad, de ah que la radioterapia


haya sido la base del tratam iento de estos tumores. Actualmente, l a
quimioterapia obtiene resultados similares.
Estadio l. El tumor tericamente est l imitado al testculo. No obs

tante, se sabe que hasta un 20% de pacientes presentan micro


metstasis a ganglios retroperitoneales, que en el momento del
diagnstico no se detectan. La presencia de micrometstasis se ha
relacionado con dos factores de riesgo: tumor de ms de 4 cm y la
invasin tumoral de la rete lestis. En estos enfermos se aconseja ad
ministrar radioterapia o quimioterapia (cisplatino). Con sto, frente
a los pacientes que optaron nicamente por observacin, la recidiva
desciende del 20 al 5%.

34

Observacin
Si FR :

> 4 cm rere testis

Radioterapia

Quimioterapia

tla,lIb

la

lIa,lIb

Observacin
Linfadenectomra
(EEUUI
Quimioterapia
(UEI

Radioterapia
Quimioterapia

+++

invasin
vascular

Quimioterapia

o
linftica

SEMINOMA I NO SElIllfIOMA

Quimioterapia
Figura 26. Algoritmo de tratamiento de la neoplasia testicular

urOlogja
Masas residuales
Se define como masa residual la existencia de conglomerados adeno

MASA RESIDUAL

pticos tras tratamiento quimioterpico o radioterpico. Cuando el tu


mor primario es un semi noma y existen masas residuales, la actuacin
SEMINOMA

a segui r ser: si la masa residual es inferior a 3 cm, tiene muy pocas

NO SEMINOMA

probabi lidades de contener tumor residual y no requiere ms que ob


servacin. Pero cuando es superior a 3 cm, se debe realizar una PET
(tomografa por emisin de positrones), si existiera esta posibilidad a
nivel tcnico, pues detecta con una alta sensibilidad y especificidad la
presencia de tumor residual. Si no se dispone de una PET o si sta es
positiva, se realizar ciruga de la masa.

< 3 cm

> 3 cm

+/- PET

Observacin

-+---

Cuando el tumor primario es un tumor no seminomatoso, se debe rea


lizar exresis de dicha masa siempre, con independencia del tamao.

e ;/ "'- $

Exresis de la masa residual

50% necrosis

La histologa de estas masas residuales, una vez extirpadas, puede ser:


tejido necrtico hasta en un 50% de las ocasiones, tumor viable en un

35% teratoma

15% tumor viable

1 5% y teratoma en un 35% (stos, dejados a libre evolucin, pueden


convertirse en teratomas malignos o producir procesos compresivos
con su crecimiento) (Figura 27).

Figura 27. Algoritmo de tratamiento de las masas residuales

Casos clnicos representativos

Un hombre de 31 aos de edad consulta por la presencia de una masa palpable en


el teste derecho, de un mes de evolucin, no dolorosa. Su urlogo le realiza una
ecografa testicular, en la que se evidencia una lesin hipoe<oica, bien delimitada,
intratesticular. los marcadores tumorales a-fetoproteina y HCG son negativos. la
actitud ms correcta de, entre las siguientes, sera:
1)

2)
3)
4)
5)

Dado que los marcadores tumorales son negativos, se descarta neoplasia testicular
y requiere observacin.
Repetir la ecografa testicular en un plazo de tres meses.
Realizacin de una tomografa axial compularizada loraco-abdrnino-plvica.
Biopsia transeseralal dcl tcstculo.
Orquiectoma radical y esperar resultado del patlogo.

Qu diagnstico, entre los siguientes, es el ms probable en un nio de 8 aos con


signos inequvocos de pubertad precoz y que, en la exploracin, presenta una masa
en cl testculo derecho de 2 cm de diilmetrol

1 ) Tumor de clulas de leydig.


2) Scminoma.
3) Tumor del saco vitelina.
4) Teraloma.
5) Coriocarcinoma.
MIR 03-04, 1 45; Re 4

MIR 05-06. 107; Re: 5

35

08.
TRAS P LANTE RENAL

Orlentaclon

MIR
lo ms imJX)rtante de
este tema son los tipos de
rechazo, que se solap...n con
Inmunologla. Se debe insislir

en el rechazo agudo, que


conviene repasar con las
preguntas de aos anleriOfe5.

OJ

las causas ms frecuentes de insuficiencia renal crnica son la diabetes mellitus y las glomerulonefritis.

la clnica caracterstica del rechazo agudo es: fiebre, hipertensin y dolor en el rea del injerto.

GJ

El rechazo agudo produce oliguria, no poliuria.

8 . 1 . I nd i caciones
las dos enfermedades que ms comnmente abocan a una insuficiencia renal terminal irreversible, tratable
mediante un trasplante renal, son la glomerulonefritis y la diabetes mellitus insuli nodependiente.
Otras causas importantes son:
Poliquistosis renal.
Nefroesclerosis hipertensiva.
Enfermedad de Alport.
Nefropata IgA.
lupus eritematoso sistmico.
Nefroesclerosis.
Nefritis intersticial.
Pielonefritis.
Uropata obstructiva.
los mejores receptores son individuos jvenes cuyo fallo renal no se deba a una enfermedad sistmica que pue
da daar el rin trasplantado o causar la muerte por causas extrarrenales. Generalmente se suele mantener al
receptor en tratamiento con dilisis durante un cierto tiempo previo al trasplante.

8.2. Contra i n d icaciones


las contraindicaciones absolutas son las siguientes:
Infeccin activa.
Enfermedad maligna que no pueda ser erradicada.
Sospecha de no cumplimiento teraputico del protocolo inmunosupresor.
Glomerulonefritis activa.
Expectativa de vida reducida por enfermedad de base del paciente.
Presencia de anticuerpos preformados frente a antgenos del donante.
7

Preguntas

- MIR 02-03, 178


- MIR 99-00F, 1 4 1

36

En referencia a las contraindicaciones relativas del trasplante renal se debe decir que stas se han ido modifi
cando a lo largo de los aos, al mejorar la tcnica y los cuidados prequirrgicos y postquirrgicos. En muchas
ocasiones, el trasplante plantea menos riesgo que una hemodilisis crnica.

urOloga

Complicaciones tcnicas. Complicaciones vasculares, hemorragia,

Actualmente se consideran contraindicaciones relativas la edad avan


zada, la oxalosis, la amiloidosis, la enfermedad iliofemoral oclusiva,

hipertensin por estenosis de la arteria renal, trombosis venosa,

las anomalas del tracto urinario inferior O las alteraciones psiquitricas


graves (MIR 99-00F, 1 4 1 ).

complicaciones del tracto urinario, necrosis tubular aguda, linfo


celes.
Complicaciones no tcnicas. Infecciones bacterianas y oportunis

tas en relacin con la inmunosupresin, hiperglucemias, compli


caciones gastrointestinales, h i perparatiroidismo y tumores (cncer

8.3. Com p l i caciones

de piel y de labios, carcinoma in silU de crvix, l infomas no

Hodgkin; guardan relacin con el tratamiento inmunosupresor).


Puede aparecer h ipertensin debida a enfermedad en los riones
originales, como consecuencia de rechazo, por estenosis de l a

Las complicaciones que se pueden presentar son las siguientes:


Rechazo (Tabla 1 8) (MIR 02-03, 1 78).

anastomosis d e l a arteria renal o por toxicidad renal por ciclos

Recurrencia de la enfermedad en el rin trasplantado.

porina.

Minutos. das

HIpoIagudD

--

Agudo
CnInk:o

Das

Semanas
Meses. aos

PA

PATOGENIA

INICIO

RECHAZO

Ac. preformados

Trombosis microvascular

CID

Isquemia o infarto

Act. del complemento

PMN en capilares

Dao endoletelial

Vasculitis necrotizante

Celular (+Ac)

Respuesta 2. a Ag-HLA

Celular (+Ac)

Infiltrado de linfocitos

Forma celular: tubulointersticial

Humoral y celular

Intima arterial aumentada

Atrofia tubular

Glomerulopata

Forma vascular: mediada por Ac

TRATAMIENTO

Nefrectoma del injerto

Bolos de esteroides

Ac monoclonales

Bolos de esteroides (la vascular suele ser resistente)

Ac monoclonales

No hay; control de HTA

Tabla 18. Rechazo en el trasplante renal

Un paciente de 35 aos, con insuficiencia renal crnica, secundaria a pielonefritis


crnica recibe un trasplante renal de cadver con el que comparla dos idenlidades
en A y 8 Y una en DR. Recibe tratamiento inmunosupresor con cidosporina A y cor
ticoides en dosis estndar. En el posoperatorio inmediato se observa buena diuresis, y
no es necesario el tratamiento sustitutivo con hemodilisis. En el 5." da de evolucin,
el paciente pt"esenta fiebre de 38", lA de 180/110, oliguria y disminucin en la con
centracin urinaria de sodio. El diagnstico ms probable sera:

1) Crisis hipertensiva.
2) Infeccin respiratoria.
3) Pielonefritis aguda del injerto renal.
4)

Recidiva de su enfermedad renal.

5) Rechazo agudo del injerto renal.


MJR 02-03, 1 78; RC: 5

37

09.
UROPATA OBSTRUCTIVA

Oflentaclon

MIR
Tema poco preguntado hasta
la fecha. Se debe tener una
idea general y aprender los
Aspectos esenciales.

GJ

la uropata obstructiva puede producir insuficiencia renal si no se resuelve a tiempo.

(TI

El dolor suele estar presente en la obstruccin aguda. Sin embargo, en la crnica, es frecuente su ausencia.

(l)

Despus de resolver una uropata obstructiva puede producirse una fase de poliuria.

9 . 1 . Caractersticas
Detencin del flujo d e orina e n cualquier punto entre los clices renales y e l exterior (Tabla 1 9).
Su i mportancia reside en el desarrollo potencial de insuficiencia renal, por lo que tiene importancia la obs
truccin urinaria bilateral o la unilateral sobre rin nico funcionante.
Una obstruccin de ms de un mes
de duracin puede dar lugar a u n

RECUERDA

dao renal funcional y estructural

la uropata obstructiva puede producir glomerulonefrltis focal y


segmentarla.

permanente.

INTRAlUMINAl

INTRAPARIETAl

litiasis
Tumores (hipernefroma,
uroteliomal
Necrosis papilar
Cogulos

Estenosis congnita
Estenosis postinfecciosa
Estenosis postraumtica
Estenosis isqumica

EXTRAPARIETAL
(compreslon extnnseca)

Urter retrocavo
RiMn en herradura
Fibrosis retroperitoneal
. Tumores (prstata,
vejiga, ginecolgicos)
Hiperplasia prosttica
ligadura iatr6gena
de urteres

EXTRAPARIETAl
(dlsfunclon neuromuscular)

Vejiga neur6gena
Vejiga automtica: lesin sobre
L1-L3

Vejiga tona: lesin bajo ll-L3


Disfuncin de la unin
pieloureteral
Reflujo vesicoureteral

Tabla 19. Clasificacin de la uropatfa obstructiva

9.2. C l n ica

(MIR 98-99F, 1 38)

La forma de presentacin depende de los siguientes factores:


Etiologa de la obstruccin. Presenta la clnica propia de la enfermedad de base.
Tiempo de evolucin. La aguda suele cursar con dolor (clico nefrtico), siendo la crnica ms frecuentemen

te asintomtica.
Lugar de obstruccin:

Preguntd<;

MIR 07..Q8, lOS


. MIR 98-99f, 138

38

Tracto urinario inferior (uretra y vejiga). Cursa con retraso para iniciar la miccin, disminucin de fuerza
y del tamao del chorro, goteo terminal, hematuria, escozor al orinar, orina turbia, retencin aguda de
orina o i ncontinencia paradj ica ("miccin por rebosamiento").
Tracto urinario superior (urter y rin). Estos pacientes presentan dolor en el flanco (rin y urter proxi
mal), dolor en flanco con irradiacin a genitales (urter medio) o sndrome miccional (urter terminal) .

urologia

Despus de resolverse una obstruccin, sobre todo si es crnica, puede

y su reversibil idad. En los casos en que hay destruccin irreversible de

producirse una fase de poliuria. Esto se debe a que, a nivel tubular,

la va urinaria, es necesario realizar una derivacin urinaria definitiva

cuando se ralentiza crnicamente el flujo urinario, se genera una in

(Figura 28).

sensibilidad a la ADH transitoria (diabetes inspida nefrognica), de ah


la poliuria.
MANEJO DE LA UROPATIA OBSTRUCTIVA

9.3. D i a g nstico
El diagnstico d e seguridad, la valoracin d e la evolucin, y el pro
nstico son ecogrficos. Adems, son tiles la anamnesis y la explo
racin fsica, la radiologa simple (conveniente en litiasis radioopaca),

.
INFRAVESICAl
(globo vesical)

SUPRAVESICAl
(no globo vesical)

SONDA VESICAL
O ClSTOSTOMIA

urografa i ntravenosa (confirma una posible anomala funcional y til


en litiasis radiotransparente), cistografa, estudio metablico (til en
prevencin de recidivas), Te abdominal, ecografa transrectal, biopsia

prosttica dirigida, pielografa retrgrada, nefrostografa, cistoscopia,


flujometra, cistomanometra y citologa urinaria (MIR 07-08, 105).

9.4. Trata m iento


Es necesario restablecer el flujo urinario. La mayor parte de las veces
se realiza mediante litotricia o correccin quirrgica. Si la obstruccin

No dilatacin

Estudio mdico

Dilatacin
bilateral

Dilatacin unilateral

Birreno

Estudio (UIV, TC)

Monorreno

Fracaso renal

Derivacin de va
urinaria superior
(nefrostomfa/doble J)

es aguda y/o bilateral, la desobstruccin es urgente, y puede lograrse


mediante un sondaje vesical, talla vesical, catter ureteral o nefrosto
ma. En caso contrario, hay que valorar el grado de sufrimiento renal

Figura 28. Procedim;ento de actuacin frente a la uropatla obstructiva

39

1 0.
DISF UNCiN ERCTIL

Orl(>ntaclon

MIR
Tema de reciente
introduccin, sencillo
y muy rentable. Se debe
incidir en los factores de
riesgo, el tratamiento
y sus contraindicaciones.
Es conveniente conocer muy
bien las preguntas aparecidas
con anterioridad.

(jJ

la causa ms frecuente de disfuncin erctil es vascular.

la enfermedad endocrina ms relacionada con ella es la diabetes mellitus.

lIJ

El sildenafilo est contraindicado en pacientes que toman nitratos o frmacos donadores de xido ntrico, en

pacientes con infarto agudo de miocardio (lAM) en los ltimos seis meses, y en pacientes con insuficiencia
cardaca grave o angina inestable.

1 0. 1 . I ntrod uccin
La disfuncin erctil (DE) se define como l a incapacidad persistente o recurrente para conseguir o mantener l a sufi
ciente rigidez del pene que permita una relacin sexual satisfactoria. Debe tener una duracin mnima de tres meses.

1 0.2. Preva lencia


En Estados Unidos, en u n estudio en varones de entre 40 y 70 aos, se estim que l a prevalencia global era del
52%. La prevalencia en Espaa se estima en 1 .500.000 a 2.000.000 varones, en torno al 1 2, 1 %.

1 0.3. Etiologa
Se puede clasificar en (MIR 04-05, 1 08):
Orgnica. Causas vascu lares (las ms frecuentes 60-80%), neurolgicas ( 1 0-20/o), hormonales (5-1 0%) o
locales.
Psicgena.
Mixta. En la mayora de los casos de etiologa orgnica se aade un componente psicolgico.

1 0.4. Factores de riesgo


Edad: factor independiente.
Diabetes: es la enfermedad endocrina ms frecuente asociada a disfuncin erctil. Significa una probabilidad tres

veces superior de presentar DE. Estn implicados mecanismos vasculares, neuropticos y disfuncin gonadal.
Enfermedad cardiovascular: cardiopata, hipertensin arterial, enfermedad vascular perifrica y descenso del
7

Preguntas

- MIR 06-07, 97
- MIR 05-06, 108
- MIR 04-05, 108

40

colesterol HDL se han relacionado de manera clara con la d isfuncin erctil.


Tabaquismo: factor independiente.
Secundaria a frmacos: aquellos que causan hiperprolactinemia, que disminuyen los niveles de testosterona,

psicotropos y antihipertensivos.

urologia

Secundaria a consumo de drogas: cocana, herona, etc.

un inhibidor de la fosfodiesterasa tipo 5 (PDE5). Induce la relaja

Trastornos afectivos: depresin.

cin del msculo liso del cuerpo cavernoso, l iberando xido ntrico
(NO). El NO liberado por el endotelio vascular y por las terminacio
nes nerviosas no adrenrgicaslno colinrgicas es el principal neu
rotransmisor de la ereccin. Precisa de deseo sexual y estimulacin

1 0. 5 . Diag nstico

previa para su efecto.


Las contraindicaciones absolutas de sildenafilo son:
Administracin concomitante con nitratos o frmacos donadores

El diagnstico debe basarse en los siguientes componentes:

de xido ntrico por el riesgo de hipotensin grave (dinitrato/mo

Historia clnica y sexual: investigar los posibles factores de riesgo

implicados.

nonitrato de isosorbide, molsidomina, nicorandil, nitroglicerina,

Exploracin fsica: encaminada a descartar enfermedad vascu

lar, enfermedades neurolgicas, trastornos genitales y endocri

nitroprusiato sdico) (MIR 06-07, 97; MIR 05-06, 108).


Pacientes en los que est desaconsejada la actividad sexual (an

nopatas. En varones mayores de 50 aos se incluir tacto rectal.

gina inestable, insuficiencia cardaca o i nfarto reciente, hace

Determinaciones analticas: glucemia basal, perfil l ipdico, testos

menos de seis meses).

terona total y l ibre y prolactina. Adems, es conveniente solicitar


Apomorfina: agonista dopaminrgico que acta a nivel central so

hemograma, funcin renal y heptica.

bre el mecanismo de la ereccin. Est contraindicado en sujetos que

Pruebas especializadas: nicamente en ocasiones muy seleccionadas.

tengan desaconsejada la actividad sexual.

1 0.6. Trata m i e nto

Seg unda lnea

Se puede estructurar en tres escalones o etapas, que sern superadas en

Terapia intracavernosa: alprostadil (PGE1), mediante inyeccin di

funcin de fracaso del escaln previo.

recta en 105 cuerpos cavernosos. Otros frmacos son la papaverina


y la fentolamina.

Frmacos orales

Tercera lnea

Citrato de si ldenafilo: se considera actualmente como el tratamien


to farmacolgico de eleccin en la DE. Hoy en da existen nuevos

Ciruga de revascularizacin (venosa, arterial). Implante de prtesis

frmacos basados en l, como vardenafilo y tadalafilo. Se trata de

de pene.

Paciente de 6J aos, en tratamiento a demanda con citrato de sildenafilo por presen


tar disfuncin erctil de aos de evolucin. Seale cul de los siguientes frmacos

NO asociara en ningn caso a su tratamiento:


1 ) Amiodarona.

J) Digoxina.
4)

5)

lndapamida.
Mononitrato de isosorbide.

MJR 05-06, 108; RC: 5

2) Verapamilo.

41

También podría gustarte